Wissen wir, warum es in unserem Universum ein Tempolimit gibt?

Bei dieser Frage geht es darum, warum wir eine universelle Geschwindigkeitsbegrenzung haben (die Lichtgeschwindigkeit im Vakuum). Gibt es ein grundlegenderes Gesetz, das uns sagt, warum das so ist?

Ich frage nicht, warum das Tempolimit gleich ist C und nicht etwas anderes, sondern warum es überhaupt eine Grenze gibt.

BEARBEITEN: Antworten wie "Wenn es nicht so wäre ..." und Antworten, die die Folgen einer Geschwindigkeitsbegrenzung erklären oder nicht, geben meiner Meinung nach keine Antwort darauf, ob es einen grundlegenderen Weg gibt, die Existenz abzuleiten und zu erklären der Grenze.

Es ist nicht universell. Es ist nur eine Grenze für das lokale Gesetz der speziellen Relativitätstheorie.
Die Erweiterung des Raums hat keine Geschwindigkeitsbegrenzung. Teilchen, die sich in der Raumzeit bewegen, haben eine Geschwindigkeitsbegrenzung von einer beliebigen Zahl c. (Andere haben erklärt, warum c endlich sein muss)
Alle Antworten sind im Grunde Antworten wie "wenn es nicht wäre ...", denn alle unsere Theorien bauen auf Postulaten auf, die im Grunde solche Argumentationen sind. Das heißt, selbst Argumente, die Mathematik verwenden, um etwas zu zeigen, stützen sich grundlegend auf Argumente wie dieses.
"Warum es eine Geschwindigkeitsbegrenzung gibt" ist eine philosophische Antwort und kann nicht von der Physik beantwortet werden. „Warum es eine Geschwindigkeitsbegrenzung geben sollte“ ist eine Frage, die sich auf unsere philosophische Interpretation bezieht, dass unser Universum deterministisch ist (wir brauchen einen Weg, um Kausalität zu verhindern). Die Lichtgeschwindigkeit, auch bekannt als "C" oder 1ls/s. Ist eine Geschwindigkeitsbegrenzung in Einsteins Relativitätstheorie axiomatisch auferlegt, um die Kausalität zu drücken. Wenn wir also die Kausalität in der Relativitätstheorie unter Druck setzen wollen, sollten wir eine universelle Geschwindigkeitsbegrenzung auferlegen, die sicherstellt, dass Ereignisse in kausaler und deterministischer Reihenfolge ablaufen.

Antworten (34)

Stellen Sie sich vor, es gibt eine Person, die es vorzieht, den Geldbetrag auf seinem Bankkonto mit dem Wert zu messen v . Die Gleichung ist v = C Tanh N , Wo N ist der tatsächliche Geldbetrag in Dollar. Diese Person wird auch verwirrt sein:

Warum gibt es eine Grenze ( C ) auf den Geldbetrag, den ich haben kann? Gibt es ein Gesetz, das besagt, dass der Wert meines Geldes v , kann nicht mehr sein als C ?

Die Antwort ist, dass er nur eine "falsche" Variable verwendet, um sein Vermögen zu messen. v ist nicht additiv – es ist eine Transformation einer additiven Variablen, N , die er verwenden muss, damit alles einen Sinn ergibt. Und es gibt kein "Gesetz des Universums", das den Wert begrenzt v – eine solche Grenze ist nur ein Produkt seiner eigenen Sturheit.

Dasselbe gilt für die Geschwindigkeitsmessung – sie ist die „falsche“ Variable, um die Bewegungsgeschwindigkeit zu beschreiben; Geschwindigkeit ist nicht additiv. Die „korrekte“ Variable heißt „ Schnelligkeit “ – sie ist additiv und es gibt keine Begrenzung dafür.

Kommentare sind nicht für längere Diskussionen gedacht; diese Konversation wurde in den Chat verschoben .
Diese Antwort gefällt mir nicht ganz. Wenn ich die Zeit messe, die das Licht braucht, um zum Mond und zurück zu gehen, kann ich messen, dass die Entfernung / Zeit ist c. Das ist also die Definition von Geschwindigkeit. Es ist ein endliches Maß. Das Extrahieren eines unendlichen Werts aus diesen beiden einfachen Messungen von Dauer und Entfernung erscheint bestenfalls künstlich.
@ njzk2 Sie sollten das Messen der Entfernung näher erläutern. Es ist nicht so "einfach", wie Sie vielleicht denken.

Ich erinnere mich, dass es einen wunderbaren Artikel gelesen hat, der nur grundlegende Algebra verwendet, um die allgemeinste Form der Formel zum Hinzufügen von Geschwindigkeiten zu bestimmen, die nur auf allgemeinen Prinzipien der Symmetrie basiert (was hier funktioniert, funktioniert auch dort usw. ) .

Ich kann das nicht finden, aber es ist einfach, Nothing but Relativity zu finden . Und andere, die von einer ursprünglichen Version von Mermin abgeleitet sind.

Ein weiterer Artikel mit der gleichen Idee (aber anderen spezifischen Axiomen) ist One more derivation of the Lorentz transformation von Jean-Mark Lévy-Leblond, veröffentlicht 1976 (danke bdforbs ).

Am Ende zeigt es, dass die bekannte spezielle Relativitätstheorie die unausweichliche Antwort ist. Im Papier ein willkürlicher nicht ermittelter Wert Z kam dabei heraus. Es gibt 3 Fälle: Negativ hat nicht funktioniert (in der Arbeit, an die ich mich erinnere, erstickt die Mathematik. In der verlinkten Arbeit ist es „nicht selbstkonsistent“, aber Greg Egan hat diesen Fall im Detail ausgearbeitet ). 0 gibt Galileos feste absolute Zeit an, und jeder positive Wert ergibt eine spezielle Relativitätstheorie mit einer Geschwindigkeitsbegrenzung.

Die Geschwindigkeit ist einfach . In natürlichen Einheiten ist es ein Wert von 1 . Der Grund, warum die Lichtgeschwindigkeit (oder irgendein masseloses Ding) etwas Spezifisches zu sein scheint, ist ihre Beziehung zu anderen Dingen. Am Ende können Sie feststellen, dass die als Feinstrukturkonstante bezeichnete Beziehung einen bestimmten Wert hat.

Ihre eigentliche Frage ist: Warum ist die Feinstrukturkonstante der Wert, der sie ist? Die Antwort ist unbekannt. Es kann durch ein tieferes Regelwerk bestimmt werden, als wir es jetzt wissen, es kann aus der Physik stammen, die wir nicht kennen, oder es kann eher ein reiner Zufall sein, wie die Anzahl der Planeten in unserem Sonnensystem, als ein Gesetz.


Also warum ist Z nicht null? Nun, wenn es sich um einen zufälligen Wert handelt, hat er eine verschwindend kleine Chance, genau null zu sein, und der tatsächliche Wert, den er hat, solange er nicht genau null ist, setzt nur einen Skalierungsfaktor und hat keine wirkliche Bedeutung.

Der Reichtum des Universums ist auf emergente Effekte der Grundlagen zurückzuführen: Da die Zeit relativ ist, bedeutet die Kombination mit der Quantenmechanik, dass Antiteilchen existieren müssen , zusammen mit Paarbildung und Vernichtung.

Wenn das Universum keine spezielle Relativitätstheorie, sondern eine feste absolute Zeit hätte, wäre es ganz anders und wir hätten überhaupt nicht das Gleiche. Es passt alles zusammen, und im Großen und Ganzen, wenn Sie sich ein tiefes Merkmal ansehen, stellen Sie fest, dass es notwendigerweise so auf den anderen tiefen Merkmalen basiert. Es geht um alles oder nichts: Sie können die endliche Geschwindigkeit der Kausalität an sich genauso wenig bestimmen, wie Sie fragen können, warum eine Seite eines Dreiecks die spezifische Länge hat, die sie hat.


Anmerkungen

Das verlinkte Papier und andere verwandte (von Mermin?) Verwenden Sie einen Wert K auf eine etwas andere Weise als das Z in dem Papier, das ich jetzt nicht finden kann. Diese leicht zu findenden verwenden auch Kalkül und Grenzen, was für diesen Zweck nicht so zufriedenstellend ist wie die alleinige Verwendung von Algebra mit vier angenommenen Beziehungen aufgrund von Symmetrie.

Der, an den ich mich erinnere, ursprünglich verwendet (IIRC)

  1. Raum ist hier und da gleich
  2. Wenn A sieht, wie sich B mit Geschwindigkeit X bewegt, sieht B, wie A sich mit Geschwindigkeit X in die entgegengesetzte Richtung bewegt. Äquiv. zum Raum ist in einem eindimensionalen Modell in jeder Richtung gleich.
  3. Ergebnisse von Experimenten sind jetzt die gleichen wie Ergebnisse von Experimenten zu unterschiedlichen Zeiten. (wie Nr. 1, aber für T statt X)
  4. wie #2, aber für T statt X

Ich denke, das ist zufriedenstellender als das Lévy-Leblond-Papier, das die universelle Zustimmung zur Kausalität als Axiom annimmt.

Das ist eine Art anthropischer Ansatz, aber es ist nicht weniger wahr. Es scheint kontraintuitiv, aber nur, weil wir die Dinge in einem kleineren Maßstab sehen, wo die Regeln anders zu sein scheinen. Es wäre, als würde man fragen, wie Menschen auf der Unterseite der Erde haften bleiben können, wenn die Erde rund ist. Diese Argumentation kommt von dem, was wir intuitiv verstehen, aber das bedeutet nicht, dass dasselbe in größerem Maßstab gilt.
Ausgehend von Symmetrieüberlegungen können Sie zeigen, dass Sie zwar nur die spezielle Relativitätstheorie als mögliche Antwort haben, aber c = unendlich nicht ausgeschlossen ist
@Neil Richtig, aber das Prinzip, das wir in diesem Fall intuitiv verstehen, ist Kausalität: Es ist ein bisschen schwierig, ein nichtkausales Universum zu verstehen, was entstehen würde, wenn das negative Vorzeichen gewählt würde. Darüber hinaus ist dies nicht der einzige Effekt, und all die seltsamen Effekte, die ein Riemannsches Universum zeigen würde, werden in Greg Egans „Orthogonal“-Trilogie untersucht. Egan gibt hier eine Zusammenfassung einiger physikalischer Aspekte .
Ich denke, das wunderbare Papier, das Sie erwähnen, ist, wie Galileo die spezielle Relativitätstheorie abgeleitet haben könnte .
@Ruslan nein, das ist es nicht. Es winkt „Symmetrie verwenden“, während es bei dem, an das ich mich erinnere, nur um die Verwendung von 4 Identitäten (Übersetzung und Reflexion in Raum und Zeit) ging, um die Funktion zu erhalten.
Mermin hat das nicht ins Leben gerufen. Es geht bis zu einem Aufsatz von Ignatowsky aus dem Jahr 1911 zurück.
Ist es "eine weitere Ableitung der Lorentz-Transformation"? doi.org/10.1119/1.10490
„Nun, wenn es irgendein zufälliger Wert ist, hat er eine verschwindend kleine Chance, genau null zu sein.“ Ich bin mir nicht sicher, ob das ein gutes Argument ist. Eine „Menge“, die Null ist (dh in gewissem Sinne „nicht existent“) in einer Theorie, scheint sie sparsamer zu machen und daher, nach Occams Rasiermesser, a priori wahrscheinlicher zu machen.

Die beste Antwort, die mir einfällt, lautet: „Weil das Universum ansonsten grundsätzlich unberechenbar wäre.“

Wir können uns die Raumzeit als vierdimensionale Mannigfaltigkeit vorstellen M ; die Gesetze der Physik bestimmen dann, wie sich Materie und Energie auf dieser Mannigfaltigkeit verhalten. (Aus Gründen der Argumentation können Sie dies als einfachen alten flachen Minkowski-Raum betrachten, obwohl das Argument verallgemeinert wird, um auch gekrümmte Raumzeiten einzuschließen.) Wir können dann die folgende Frage stellen: „Angenommen, ich weiß, wie sich Materie und Energie verhalten einen endlichen Teil des Universums zu einem bestimmten Zeitpunkt T = 0 . Was sagt mir das über das Verhalten von Materie und Energie im Universum nach dieser Zeit?"

Wenn es eine Geschwindigkeitsbegrenzung für das Universum gibt, dann gibt es eine Region der Raumzeit, die als Abhängigkeitsbereich bezeichnet wird , in der man vorhersagen kann, was nach unserem Anfangsmoment passieren wird. Es besteht aus allen raumzeitlichen Ereignissen, auf deren vergangene Lichtkegel 1 zurückgeführt wird T = 0 , sind vollständig in der Region des Weltraums enthalten, von der wir Kenntnis hatten. Als Funktion der Zeit betrachtet, schrumpft der im Abhängigkeitsbereich liegende Raumbereich allmählich zu nichts zusammen, da sich Einflüsse von außerhalb unseres ursprünglichen Bereichs (von dem wir keine Kenntnis hatten) nach innen ausbreiten. Aber wenn das Universum an allen Punkten eine Geschwindigkeitsbegrenzung hat, dann ist uns garantiert, dass es ein begrenztes Raumzeitvolumen gibt, in dem wir vorhersagen können, was passieren wird.

Wenn es aber eine unendliche Ausbreitungsgeschwindigkeit des Universums gibt, dann verschwindet der Abhängigkeitsbereich. Grob gesagt können wir nichts vorhersagen, da sich kausale Einflüsse von außerhalb unseres Bereichs der ursprünglichen Daten ausbreiten und im nächsten Moment alles durcheinander bringen könnten T = 0 . Wenn es also keine Geschwindigkeitsbegrenzung gäbe, wäre das Universum im Grunde unberechenbar; Ohne Kenntnis von allem , was zu einem bestimmten Zeitpunkt im Universum vor sich geht, hätten die Gesetze der Physik keine Vorhersagekraft.

Ich gebe offen zu, dass dies nicht so sehr eine Antwort auf "Warum gibt es eine Geschwindigkeitsbegrenzung?" als "Wie würde das Universum aussehen, wenn es keine Geschwindigkeitsbegrenzung gäbe?" Dennoch ist ein Universum ohne Geschwindigkeitsbegrenzung ausreichend fremd und unverständlich, um mich froh zu machen, dass ich in einem Universum mit einem lebe. (Dies hat Schattierungen des anthropischen Arguments – vielleicht argumentiert in einem Paralleluniversum eine unverständliche Kreatur darüber, wie schrecklich es wäre, in einem Universum mit einer Geschwindigkeitsbegrenzung zu leben.)

Beachten Sie schließlich, dass nichts in diesem Argument auf der speziellen Relativitätstheorie beruht; alles, was erforderlich ist, ist, dass es an jedem Punkt im Raum eine Vorstellung von einem "Lichtkegel" gibt. Die Geschwindigkeitsbegrenzung könnte von Punkt zu Punkt variieren oder sich mit der Richtung unterscheiden, aber solange sie die Nachbarschaft jedes Raumzeitpunkts in eine kausale Vergangenheit, eine kausale Zukunft und kausal nicht verbundene Regionen trennt (wie bei herkömmlichen Lichtkegeln in der konventionellen Relativitätstheorie). , dann folgt noch das Argument.


1 „Lichtkegel“ bedeutet hier nicht unbedingt „der zeitlich zurückverfolgte Weg aller Lichtstrahlen“, sondern vielmehr „zeitlich zurückverfolgte Wege aller mit der Geschwindigkeitsbegrenzung reisenden Strahlen“.

Solange die Stärke der Effekte mit der Entfernung gegen 0 geht, sehe ich keinen Grund, warum diese Idee des "Bereichs der Abhängigkeit" eine Notwendigkeit ist. Wir können bereits nicht genau vorhersagen, was in einem endlichen Raumvolumen passiert, und wir sind damit einverstanden, also warum sollte es etwas ändern?
@immibis: Es gibt keinen Grund zu erwarten, dass die Stärke der Effekte auf 0 geht. Wenn Sie sich alle Dinge ansehen, die Sie innerhalb von "einem Takt" betreffen könnten, würden Sie diese weiter entfernt erwarten (was müsste sich schneller bewegen, um dich zu erreichen), um quadratisch in Stärke vs. Distanz zu wachsen.
Diese Antwort ist tief. gut.
@R.. Elektromagnetische Strahlung und Schwerkraft zerfallen beide 1 R 2 - Der quadratische Zerfall hebt die quadratische Menge an Dingen auf, die Sie beeinflussen könnten, sodass die Dinge, die 500 Milliarden km entfernt sind, Sie nicht stärker beeinflussen können als die Dinge, die 5 cm entfernt sind. (Der Gesamteffekt ist theoretisch immer noch unbegrenzt, aber in der Praxis wird der 500 Milliarden km entfernte Raum zu einem viel geringeren Anteil ausgefüllt.)
Stattdessen Ding eines Teilchens.
Ist das Universum nicht bereits grundsätzlich unberechenbar? Radioaktiver Zerfall: Sie haben keine Ahnung, wann genau ein einzelnes Atom zerfallen wird. Wenn Sie eines davon in Ihrem beobachtbaren Teil des Weltraums haben, können Sie nicht vorhersagen, wie es bei t = 1 aussehen wird, da es möglicherweise zerfallen ist oder nicht. Mit einer unendlich kleinen Wahrscheinlichkeit könnte Ihr ganzes Experiment einfach explodieren. Dasselbe gilt für Quantenvakuumfluktuationen ...
Andererseits sehe ich praktisch kein Problem, wenn das Universum theoretisch unberechenbar ist. In fast allen unseren realen Simulationen und Vorhersagen glätten wir unwahrscheinliche externe Ereignisse. Wenn ich vorhersagen möchte, wo der Mond morgen steht, berücksichtige ich nicht, dass ihn jemand in der Zwischenzeit sprengen könnte, auch wenn dies mit Unterlichtgeschwindigkeit möglich ist. Das wäre dasselbe - natürlich könnte jederzeit etwas extrem Schnelles auftauchen und alles verändern, aber es würde so viel Energie brauchen, um so schnell zu werden, dass es nicht wirklich jederzeit passieren würde ...
@immibis In vernünftigen Maßstäben scheint das Universum einheitlich zu sein. Also trägt die Millionen Lichtjahre dicke Universumshülle bei 1 Milliarde Lichtjahren, 10 Milliarden Lichtjahren, 100 Milliarden Lichtjahren, 1000 Milliarden Lichtjahren, 10000 Milliarden Lichtjahren, 100000 Milliarden Lichtjahren alle die gleiche Menge bei. Die erste dieser Hüllen erzeugt ein einzelnes Photon, das Ihren Körper im Laufe Ihres Lebens trifft. Nun, es gibt unendlich viele solcher Muscheln. Und etwas mal unendlich wird deinen Tag ruinieren.
Interessante Perspektive. Das scheint darauf hinzudeuten, dass der einzige Grund, warum Zeit nicht absolut ist, darin besteht, dass am Anfang des Universums beliebig viele Objekte stehen würden, die nicht die Ursache von irgendetwas anderem sind, während wirklich jede Wirkung das Ergebnis einer einzigen Initiale sein kann "Ursache". Wenn ein Teich mit Wellen und Lärm übersät ist, ist es sinnvoll anzunehmen, dass er mit einem einzelnen Stein begann, der hineinfiel, nicht dass es schon immer so gewesen wäre.
was meinst du mit "vorhersagen"? Wir können die Zukunft im wirklichen Leben nicht vorhersagen, also verwirrt mich das
Die unverständliche Kreatur stellt fest, dass das Wissen über alles, was zu einem bestimmten Zeitpunkt im Universum passiert ist, viel einfacher ist, wenn es keine universelle Geschwindigkeitsbegrenzung gibt.

Physik ist eine wissenschaftliche Disziplin, in der Beobachtungen und Messungen mit mathematischen Modellen versehen werden, die vorhandene Daten beschreiben und neue Werte für neue Randbedingungen erfolgreich vorhersagen. Wenn dies geschieht, sagt man, dass das Modell validiert wurde.

Sollten neue Experimente und Beobachtungen das Modell falsifizieren, muss man die Annahmen erneut überprüfen und sogar nach einem neuen Modell suchen.

Als validiertes Modell für Elementarteilchen haben wir derzeit das Standardmodell , das relativistische Quantenmechanik verwendet und unzählige Male mit Labor- und Beobachtungsexperimenten getestet wurde. Dieses mathematische Modell stimmt, da es die spezielle Relativitätstheorie beinhaltet, mit der Beobachtung überein, dass die Lichtgeschwindigkeit im Vakuum eine Konstante c ist. Es stimmt, der Wert von c ist für diese Diskussion zufällig. Es ist die Existenz der Grenze, die in Frage gestellt wird, und die einzig mögliche Antwort ist: weil das theoretische Modell mit dem Experiment übereinstimmt und sehr prädiktiv ist.

Wenn neue Daten das Standardmodell so verfälschen, dass ein neues theoretisches Modell notwendig wird, muss dieses neue Modell für die Fälle, in denen es validiert wurde, die bestehende Struktur einschließlich der Lichtgeschwindigkeitsgrenze übernehmen. Das Standardmodell würde zu einem Grenzfall für die neue Theorie, für die validierten Energien und Randbedingungen, ähnlich wie die Newtonsche Physik aus der speziellen Relativitätstheorie an der Grenze niedriger Energien hervorgeht.

@annav also, das Fazit ist, dass die grundlegendste Erklärung, die wir im Moment dafür haben, ist, dass es "einfach so ist", richtig?
@LandosAdam Ja. Aus den Daten leitet sich die Theorie ab, die eine begrenzte und feste Geschwindigkeit für masselose Teilchen fordert. Die Physik beantwortet letztlich keine „Warum“-Fragen. Es sagt aus, "wie" man mit dem akzeptierten Modell zu einer Beobachtung kommen kann. Wenn wir eine variable Lichtgeschwindigkeit beobachtet hätten, hätten wir eine andere Theorie entwickelt.
@annav das weiß ich, das ist in gewisser Weise die Schönheit seiner Objektivität. Aber manchmal kann eine Theorie einige Dinge erklären, die zuvor als grundlegend angesehen wurden. Deshalb habe ich diese Frage gestellt. Und so wie ich es betrachte, macht mich die Tatsache, dass die Möglichkeit besteht, dass es aus grundlegenderen Gesetzen abgeleitet werden könnte, die wir noch nicht kennen, noch enthusiastischer. Aber das Schlüsselwort hier ist "KÖNNTE", denn es könnte genauso gut so sein, wie Sie sagten.
@LandosAdam: genau. Wenn wir fragen: „Warum bewegen sich Objekte in Kegelschnitten um/an der Sonne vorbei?“, dann kann die Physik „Warum“ sagen, und zwar in Bezug auf andere Eigenschaften des Modells, die in gewissem Sinne als grundlegender angesehen werden können: eine Umkehrung -quadratische Beschleunigung führt zu bestimmten geometrischen Merkmalen. Natürlich ist es immer noch keine "ultimative" Antwort auf das "Warum", aber es ist eine Antwort. Aber wenn es darum geht, warum Licht nicht sofort ankommt, sagt Anna hier, dass es nur eine mögliche Antwort gibt, das heißt, das Modell liefert keine "tiefere" Grundlage für das Phänomen.
Es ist auch wichtig zu bedenken, dass es etwas subjektiv ist, ob etwas eine „Warum“-Frage beantwortet oder nicht, da wir über vereinfachende Erklärungen sprechen („Vielfache von 10 enden auf 0, weil wir sie zur Basis 10 schreiben“). keine körperlichen Ursachen ("Mein Fuß tut weh, weil er weicher ist als der Stein, den ich gerade getreten habe"). So sind andere zu dem Schluss gekommen, dass es eine Antwort auf das „Warum“ gibt , und tatsächlich können sie über Merkmale des Modells sprechen, die für sie teilweise die Existenz einer Grenze im Modell erklären.
@SteveJessop Ich stimme dem zu, was Sie sagen, obwohl ich nicht verstehe, warum die Antwort auf "Was ist der grundlegendste Grund dafür, dass dieses X-Ding passiert" subjektiv ist. Es ist keine Frage der Meinung. Es scheint Ansichtssache zu sein, weil einige Leute mehr wissen als andere, und so bekommen wir viele Antworten. Aber ich bin mir sicher, dass Sie die grundlegendste Antwort auf dieser Seite finden werden, wenn Sie tiefer in die meisten Antworten einsteigen, und in diesem Fall ist, dass wir das Warum dahinter nicht kennen (falls es ein Warum gibt).
@LandosAdam: Wenn Sie nicht sehen, warum die Antwort auf „Was ist der grundlegendste Grund für X“ subjektiv ist, dann müssen Sie darüber nachdenken, was Ihre Definition von „fundamental“ ist . Ich garantiere, dass es nicht objektiv ist.
@QuanticMan hat gerade diese Diskussion gesehen. Physik verwendet Mathematik. In der Mathematik sollten Sie wissen, dass die Wahl der Axiome subjektiv ist und normalerweise von der Minimierung von Schritten abhängt. Für eine axiomatische mathematische Theorie gilt, dass man ein Axiom durch einen Satz ersetzen könnte, und dann wird aus einem Axiom ein Satz, dh aus den Axiomen beweisbar. In der Physik gibt es Postulate, die die Teilmenge der axiomatischen Mathematik herausgreifen, die zu den Daten passt. Im gleichen Sinne wie bei den Axiomen könnte man verschiedene Postulate auswählen und Mathematik verwenden, um die vorherigen Postulate zu beweisen, ein subjektiver Teil.
@annav, deine Kommentare und Beiträge sind IMMER auf den Punkt gebracht und sehr aufschlussreich. Posten Sie weiter ... Ich genieße es WIRKLICH, Ihre Antworten zu lesen.
Diese Theorie habe ich schon lange. Ich glaube, dass die Lichtgeschwindigkeit auf das begrenzt ist, was sie ist, weil der Fluss der ZEIT (in anderen Einheiten) genau gleich ist und alles, was schneller als Licht ist, die ZEIT in die Zukunft (oder nirgendwo) durchbrechen würde, um nie wieder gesehen zu werden. Auch wenn die Zukunft zum JETZT wird, bewegt sich das DING immer noch in die neue Zukunft des neuen JETZT. Also, wenn sich alles Licht schneller bewegen würde – es wäre KEIN Licht zu sehen und wir wären nicht hier.
@annav Du meinst, selbst wenn wir das grundlegendste Verständnis unseres Universums erreichen, könnte jemand diese Frage beantworten, indem er ein Axiom durch ein Theorem ändert und eine andere Erklärung gibt? Deshalb wird eine Erklärung eines "fundamentalen" Warum als subjektiv betrachtet? (Könnten wir dieses Gespräch auch in einem Chatroom verlängern?)
Theoretische Ideen existieren nicht in einem theoretischen Vakuum. Appelle zum Experimentieren sind nicht die einzig möglichen Antworten auf „Warum“-Fragen.
@BenCrowell Offensichtlich sind wir anderer Meinung. „Warum“-Fragen, nachdem sie „wie mathematisch sie in das theoretische Modell passen“ durchlaufen haben, enden bei den Postulaten/Prinzipien/Gesetzen, die das Destillat von Beobachtungen/Experimenten sind, zusätzliche Axiome, die der Mathematik auferlegt werden, um die Teilmenge der relevanten Lösungen zu erfassen zu Daten. Dann ist die einzige Antwort "weil das beobachtet wurde".
@annav Warum überhaupt etwas erklären? Warum überhaupt eine Theorie formulieren? Nur damit Sie sich Daten leicht merken können? Wir haben jetzt gute Computer, denke ich, also müssen wir uns nichts merken. Dann glaube ich, dass eine große Tabelle aller Beobachtungen die beste Physik ist, die wir in Ihrem Universum machen könnten. Ich bin froh, dass ich nicht in deinem Universum lebe.
Theoretische Physik ist keine Gedächtnisstütze, um sich an die experimentellen Ergebnisse zu erinnern. Zu jedem Zeitpunkt ist die grundlegendste Theorie, die wir haben, genau die Antwort darauf, warum all die Phänomene auftreten, die durch diese grundlegende Theorie erklärt werden können. Es wäre entweder dumm oder unehrlich zu sagen: "Wir können nicht erklären, warum wir ein inverses quadratisches Kraftgesetz für die Schwerkraft oder den Elektromagnetismus haben, abgesehen von der Aussage, dass wir sie so gefunden haben." Grundprinzipien der GR- und YM-Theorie sind der Grund, warum wir diese umgekehrten quadratischen Gesetze haben.
@Dvij Ich bin anderer Meinung. Wir haben zuerst das 1/r^2-Verhalten untersucht und gefunden und es dann verwendet, um zukünftiges Verhalten vorherzusagen. Datenbanken haben wie Karten keine Vorhersagefähigkeit. In der Physik ist die Möglichkeit von Vorhersagen mit mathematischen Werkzeugen (mathematische Modelle) wichtig. Korrekte Vorhersagen validieren das Tool, falsche führen zu Änderungen in den Modellen. Physikalische Theorien werden so formuliert, dass zukünftige Anordnungen vorhergesagt und die Theorie anhand neuer Daten überprüft werden können.

Maxwell führte eine rigorose mathematische Untersuchung der Eigenschaften von Elektrizität und Magnetismus durch und bewies, dass es ein Phänomen geben muss, das er elektromagnetische Wellen nannte . Nach Maxwells Theorie muss sich eine elektromagnetische Welle mit einer konstanten Geschwindigkeit ausbreiten, die er nannte C , und die aus anderen damals bekannten und messbaren physikalischen Konstanten berechnet werden konnte.

Zu dieser Zeit diskutierten Wissenschaftler aktiv über die Natur des Lichts. Nachdem Hertz in einem Laborexperiment die Existenz elektromagnetischer Wellen nachgewiesen hatte, wurde Maxwells Theorie plötzlich zur beliebtesten Erklärung für Licht.

Das Komische an Maxwells Theorie war, dass die Geschwindigkeit relativ zu dem war, der sie gemessen hat. Wenn Sie und ich beide die Geschwindigkeit von Wellen gemessen haben, die von derselben Quelle ausgehen, sollten wir beide dasselbe Ergebnis erhalten, unabhängig von unserer Bewegung relativ zur Quelle oder zueinander.

Einige Physiker fanden das beunruhigend und versuchten es auf unterschiedliche Weise zu rechtfertigen. Einsteins große Errungenschaft bestand darin, ihre verschiedenen Ideen zu einer einzigen konsistenten, strengen mathematischen Theorie zu kombinieren. Eine der Konsequenzen seiner Theorie – bewiesen in der Mathematik – ist, dass, wenn etwas eine charakteristische Geschwindigkeit hat, die für jeden Beobachter gleich sein muss, niemand jemals etwas beobachten kann, das sich schneller als diese Geschwindigkeit bewegt.

Die universelle Geschwindigkeitsbegrenzung ist also eine mathematische Folge bestimmter messbarer und, soweit jeder weiß, grundlegender physikalischer Konstanten; B. die Durchlässigkeit des freien Raums .

Ihre Frage läuft dann darauf hinaus: "Warum hat das Universum diese Eigenschaften?"

Jedes Mal, wenn Physiker eine „Warum“-Frage beantworten, beruht die Antwort immer auf tieferen Ebenen des „Warum“?

Tatsächlich zeigen die Gleichungen die Geschwindigkeit als Funktion der magnetischen und elektrischen Konstanten. Es ist eine weitere Beobachtung, dass sich diese nicht wie ein normales Menium verhalten, zu dem sich der Beobachter relativ bewegt, sondern unabhängig von Ihrer Bewegung hartnäckig den gleichen Wert ablesen.
"Das Komische an Maxwells Theorie war, dass die Geschwindigkeit relativ zu dem war, der sie gemessen hat." Müsste das nicht heißen: "war nicht relativ zu wem es jemals gemessen hat" , also unabhängig vom Bezugssystem?
Das Komische an Maxwells Theorie war, dass die Geschwindigkeit relativ zu dem war, der sie gemessen hat. - Ich glaube nicht, dass das stimmt, denn wenn es so wäre, wäre das Ergebnis des Michelson-Morley-Experiments nicht so ein Schock gewesen. Übersehe ich etwas?
die Geschwindigkeit war relativ : Er meint, dass die berechnete Geschwindigkeit c für jeden Beobachter als derselbe Wert angenommen wird. Es ist derselbe Satz Mehrdeutigkeit, der der Relativitätstheorie ihren Namen gab, da Einstein dasselbe Problem damit auf Englisch anmerkt. Die Geschwindigkeit der Welle ist c relativ zu Alice, und die Geschwindigkeit ist c relativ zu Bob.
Dies weist genau in die richtige Richtung und verdient es, weitaus höher bewertet zu werden als die aktuelle "Top" -Antwort. Ich würde es lieben, wenn Sie dies um die grundlegenden Punkte der Ableitungen (sowohl Maxwells als auch Einsteins) erweitern würden.
@Floris, ich bin hier nur ein Tourist. Ich würde gerne eines Tages etwas Physik lernen. Ich habe mehr Zeit damit verbracht, als ich zugeben möchte, mich durch „The Road To Reality“ von Roger Penrose zu arbeiten. Meine größte Erkenntnis war, dass Physik ein großes Thema ist. Dieses Buch besteht im Wesentlichen aus elfhundert Seiten mit kurzen Einführungen in einige der mathematischen Techniken und Formen, die man lernen müsste, um die Physik zu verstehen. Vielleicht, wenn (falls!) ich in Rente gehe...

In der Physik kann man das Warum nicht ohne Mehrdeutigkeit fragen/beantworten . Nun stellen wir fest, dass die Lichtgeschwindigkeit endlich ist und dass dies die höchste Geschwindigkeit für die Energie zu sein scheint.

Effektive Theorien wurden um diese Einschränkung herum aufgebaut und sie sind konsistent, da sie auf Messgeräten beruhen, die auf Technologie/Wissenschaften basieren, die alle eingebaut sind. In den modernen Wissenschaften kümmert man sich nicht darum, was passiert, sondern darum, was passiert Geräte messen.

Die Validierung dieser Theorien lässt leicht erkennen, dass es eine universelle Höchstgeschwindigkeit gibt. Tatsächlich gibt es eine maximale Geschwindigkeit für ein sich bewegendes, aus Energie bestehendes Objekt, wenn es in einem statischen Raum gemessen wird. Es ist nicht exakt in einem expandierenden Universum oder in anderen kritischen relativistischen Kontexten. Es ist nicht genau, wenn es sich nicht um Energie handelt, dh mit dem vermuteten Einfluss, der durch verschränkte Teilchen ausgetauscht würde. Trotzdem kann man nicht schneller als das Licht reisen, es sei denn in Träumen.

Die moderne Physik ist neu. Es erstreckt sich in alle Richtungen. Es ist immer noch schwierig, eine Synthese aus all dem zu machen, was bekannt ist, was nützlich ist und was nicht konsistent und relevant ist. Vielleicht wird der endliche Wert von c (und nicht nur c) aus der Quantentiefenfeldanalyse stammen, genauso wie die Relativitätstheorie der sehr reichhaltigen Maxwell-Theorie folgte. Inzwischen ist der wissenschaftlichen Gemeinschaft eine solche neue Analyse nicht bekannt, auch wenn sie bereits in einem obskuren Archiv veröffentlicht wurde.

Das ist "Woher wissen wir, dass es in unserem Universum eine Geschwindigkeitsbegrenzung gibt?" Das ist eine ganz andere Frage als "Wissen wir, warum es in unserem Universum eine Geschwindigkeitsbegrenzung gibt?"
@immibis Ich habe versucht zu zeigen, dass es keine Antwort auf dieses Warum gibt, weil es sich um eine experimentelle Tatsache handelt, die wie ein Postulat angenommen wird, auch wenn das Konzept der Geschwindigkeit diskutiert werden kann. Vielleicht wird sich sein Status mit mehr Wissen und mehr Synthese entwickeln, aber wer weiß heute, wie man die Geschwindigkeitsbegrenzung aus anderen Annahmen ableitet?
Es gibt ein Problem mit der Frage nach dem „Warum“ in den Wissenschaften. Im Alltagsenglisch sind „warum“ und „wie“ oft synonym: Die Antwort auf „Warum ist der Himmel blau“ ist die gleiche wie „Wie ist der Himmel blau“. Aber für tiefere Fragen wird das „Warum“ bedeutungslos, weil es eine teleologische Frage ist (nicht theo_logisch) – es geht letztlich um die Frage nach dem Zweck oder Endziel . Aber die Naturgesetze sind nicht auf ein bestimmtes Ziel ausgelegt; sie sind einfach. Daher ist das „Wie“ wirklich die einzige Frage, die die Wissenschaft beantworten kann. Das Wie ergibt sich in diesem Fall aus den Gleichungen.
@ user151841 ja, denn warum muss durch eine logische Demonstration in der aktuellen Theorie beantwortet werden. Aber wir können kein Postulat beweisen. Dabei bleibt man bei robusten Beobachtungen und erfahrenen Theorien
@ user151841 Obwohl ich Ihnen teilweise zustimme und dazu neige, diese Fragen in einen Teil der Metaphysik zu stellen, bin ich nicht damit einverstanden, dass Sie sie auf ein teleologisches Argument reduzieren. Es ist nicht dasselbe wie zu fragen, warum der Wert von π ist, was es ist, was bedeutungslos wäre. Vielmehr kann es hier der Fall sein, dass die Alternative nicht einmal im mathematischen Sinne konsistent ist, was eine nicht teleologisch argumentierende Antwort liefern würde.
@G.Bergeron Ich bin nicht anderer Meinung; Für ein anspruchsvolles Publikum sind Erklärungen, die Epiphänomene einfacherer Phänomene sind oder auftauchende Verhaltensweisen oder Eigenschaften sind, in der Regel zufriedenstellend für eine „Warum“ -Frage. Ich vermute, das ist es, was sie "eigentlich" fragen
Wenn die Physik uns keine Antwort auf die Frage "warum" geben will, sollten wir stattdessen die religiösen Bücher lesen, um die Antwort zu lernen?

Diese Frage hat einige interessante Antworten ausgelöst, und ich möchte auch einen Beitrag dazu leisten. Es sollte vollkommen klar sein, dass wir in einer Welt mit einer endlichen Höchstgeschwindigkeit leben, und viele Antworten haben die Folgen und Gründe dafür berührt.

Ich möchte aber auf einen Aspekt hinweisen, der bei den anderen Antworten ganz vergessen zu sein scheint. Wenn die Lichtgeschwindigkeit unendlich wäre, hätten wir überhaupt kein Licht.

Um dies zu sehen, werfen Sie noch einmal einen Blick auf die Maxwell-Gleichungen. Beachten Sie das in ihnen C = 1 μ 0 ϵ 0 , also wenn Sie festlegen C dann entweder (oder beides) von μ 0 Und ϵ 0 müsste null sein. Dies wird die Existenz dynamischer Magnetfelder effektiv beseitigen.

Besonders für Licht bedeutet es das × B = 1 C 2 E T 0 , also wären Magnetfelder statisch (und von null Intensität, denken Sie daran, dass es keine magnetischen Monopole gibt). Somit wäre vom Elektromagnetismus nur noch die Elektrostatik übrig.

Physikalisch macht das auch Sinn, wenn C dann wäre die Reaktion des elektrischen Feldes auf jede Umordnung der Ladungen augenblicklich, also gibt es keinen Platz (Zeit?) für eine Reaktion des magnetischen Feldes.

Wenn man an Magnetfelder (und insbesondere an die Lorentzkraft) denkt, ist es auch sinnvoll, dass Magnetfelder verschwinden sollten. Wenn C es gibt keine Längenkontraktion, und daher werden keine Lorentz-Kräfte auf Teilchen wirken.

Wenn wir also von Signalen sprechen, die sich unendlich schnell ausbreiten, ist es zweifelhaft, auf welche Signale wir uns beziehen.

Ich denke, deine Antwort ist die erste, die wirklich zeigt, was wann kaputt geht C tendiert gegen unendlich, ohne tatsächlich das Zirkelargument "Kausalität, weil SR, und mit unendlichem c keine Kausalität" usw. hervorzurufen. Danke und +1
Um ehrlich zu sein, ich nicht wirklich mit dem Argument, dass C bedeutet keine Kausalität. Dies ist nur die Newtonsche Grenze, bei der die Zeit absolut ist, und soweit mir bekannt ist, funktioniert dies gut.
Unendlichkeit ist viel größer als Lichtgeschwindigkeit. Warum ist die Lichtgeschwindigkeit immer und überall gleich? Weil der Raum die Dichte ändert, was durch Rotverschiebung und dergleichen beobachtet wird, aber ändert sich nicht auch die Zeit (Lichtgeschwindigkeit)?
@CeesTimmerman: Ich würde sagen, wir sind uns nicht sicher, warum C hat in der gesamten Raumzeit den gleichen Wert. Das stärkste Argument wäre das des allgemeinen Relativitätsprinzips. Ich bin mir jedoch sicher, dass, wenn die Lichtgeschwindigkeit (raumzeit-)positionsabhängig wäre, einige seltsame Effekte auftreten würden, die hoffentlich von der Erde aus erkennbar wären.
Zu sagen „Lichtgeschwindigkeit kann nicht unendlich sein“ ist keineswegs dasselbe wie „nichts kann schneller gehen“.
@Floris: Ich stimme zu, Letzteres ist eine Folge der speziellen (oder allgemeinen) Relativitätstheorie. So lange wir daran glauben, geht nichts schneller als Lichtgeschwindigkeit, nicht einmal Neutrinos ;). Die erste Aussage ist eher eine experimentelle Tatsache. Die Lichtgeschwindigkeit ist endlich. Dies hindert jedoch nicht daran, dass es im Grenzbereich in sich widerspruchsfreie Theorien gibt C . Dieser Beitrag war nur ein (von vielen) Beispiel(en), warum diese Grenze viel langweiliger ist als die für finite C .

Warum haben wir ein allgemeines Tempolimit? Gibt es ein grundlegenderes Gesetz, das uns sagt, warum das so ist?

Die grundlegenderen Gesetze sind Kausalität und Lokalität. Kausalität drückt die Tatsache (oder Annahme) aus, dass Wirkungen Ursachen nicht vorausgehen können, und Lokalität drückt die Tatsache (oder Annahme) aus, dass grundlegende kausale Beziehungen durch Differentialgleichungen beschrieben werden.

Angesichts dieser beiden Grundprinzipien schreibt die Logik der Mathematik vor, dass die Differentialgleichungen entweder parabolisch (ähnlich einer Wärmegleichung) oder symmetrisch hyperbolisch (ähnlich einer Wellengleichung) sind.

Wenn sie parabolisch sind, gibt es keine Geschwindigkeitsbegrenzung. Beispielsweise breitet sich Wärme gemäß der Wärmegleichung augenblicklich zu beliebig weit entfernten Orten aus, obwohl sie exponentiell mit der Entfernung unterdrückt wird.

Wenn sie symmetrisch hyperbolisch sind, impliziert die mathematische Theorie eine endliche Ausbreitungsgeschwindigkeit. Dies ist zum Beispiel bei den Maxwell-Gleichungen der Fall, die die Geschwindigkeit elektromagnetischer Signale auf eine Zahl namens Lichtgeschwindigkeit begrenzen.

Es ist eine experimentelle Tatsache, dass sich die Natur nach der zweiten Möglichkeit verhält - auch unabhängig von Überlegungen zur Lichtgeschwindigkeit. Es gibt überwältigende Beweise dafür, dass alle grundlegenden Prozesse in der Natur symmetrisch hyperbolischer Natur sind. Gleichmäßige Wärme - die Wärmegleichung ist nur die einfachste Näherung, bei der die Geschwindigkeitsbegrenzung verloren geht. Aber ausgefeiltere Ableitungen aus der statistischen Nichtgleichgewichtsmechanik erzeugen symmetrische hyperbolische Gleichungen, die erst bei weiterer Annäherung parabolisch werden.

Dass die Grenzgeschwindigkeit die Lichtgeschwindigkeit ist, ist sehr wahrscheinlich, aber nicht unbedingt der Fall. Es ist mit der Annahme verbunden, dass Photonen masselos sind. Wären Photonen massereich, Gravitonen aber masselos, wäre die Lichtgeschwindigkeit kleiner als die theoretische Grenze der Signalgeschwindigkeiten im Universum – die dann der Gravitationsgeschwindigkeit entspräche.

Allerdings sind laut Particle Review der Particle Data Group die Obergrenzen für die Masse eines Photons extrem klein, und die Beobachtungen stimmen derzeit voll und ganz mit der Annahme masseloser Photonen überein.

Danke schön! Ich habe nach dieser Antwort gesucht. Ich wollte fragen, ob Kausalität und Lokalität (Hyperbolizität) in irgendeiner Weise unterschiedlich sind. Wenn die Hyperbolizität eine obere Geschwindigkeitsgrenze, sagen wir die Lichtgeschwindigkeit, bereitstellt, impliziert sie auch das Verbot der superluminalen Informationsübertragung und damit Kausalität. Sie sind also dasselbe, oder?
@NanashiNoGombe: Lokalität bedeutet beschreibbar durch eine Differentialgleichung. Eine Parabelgleichung ist lokal, aber nicht kausal.
Vielen Dank für Ihre schnelle Antwort. Ich habe gefragt, ob Hyperbolizität dasselbe ist wie relativistische Kausalität? Ich denke, es ist. Bitte korrigieren Sie mich, wenn ich falsch liege.
@NanashiNoGombe: In der klassischen relativistischen Physik ist es dasselbe. In der Quantenphysik ist die Beziehung komplizierter.
Vielen Dank für diese Antwort. Bei weitem das Beste.
Hallo, könnten Sie bitte erklären oder einige Referenzen (Artikel, Bücher usw.) angeben, wie die beiden Grundprinzipien vorschreiben, dass die Differentialgleichungen entweder parabolisch oder symmetrisch hyperbolisch sind? Und welche Differentialgleichungen?
@ApoorvPotnis: Die partiellen Differentialgleichungen für klassische Felder müssen ein gut gestelltes Anfangswertproblem haben. Die Berücksichtigung kleiner Störungen mit lokalen, stark oszillierenden Wavelets schränkt die Form der charakteristischen Sätze der PDE ein. es steht wahrscheinlich irgendwo in Hörmanders 4-bändiger Abhandlung.
Vielen Dank, Arnold, für die bisher aufschlussreichste Antwort. Die Lichtgeschwindigkeit hat mit nichts anderem als Kausalität zu tun. Es ist die Geschwindigkeit, mit der alles passiert. Und es ist konstant und unveränderlich und gleich 1ls/s. Und es ist so, damit die Kausalität unter Druck gesetzt werden kann! Beste Antwort bisher, danke Sir.

Die Existenz der Geschwindigkeitsbegrenzung hängt mit der Existenz der Zeit zusammen [UPDATE: Zeit ist ein Maß, das nur verfügbar ist, wenn C ist begrenzt. Wenn Sie nicht einverstanden sind, geben Sie eine Möglichkeit an, die Zeit zu messen C ist unendlich vor dem Down-Voting]. Wenn es kein Tempolimit gäbe, würde alles sofort passieren. Auch würden irgendwelche Wellen in irgendeiner Angelegenheit nicht beeinflusst und sich momentan ausbreiten. Die Zeit würde verschwinden (wie auch die Entfernung und folglich der Raum, übrigens).

Es ist also dasselbe wie "Warum gibt es Zeit?". Die derzeit begrenzte sofortige Energieübertragung würde die Welt, wie wir sie kennen, verändern und es wäre nicht mehr diese Welt, die wir kennen. Die Newtonsche Physik würde als Konzept verschwinden, da die Materie selbst nicht mehr so ​​funktionieren würde. Sowie der Begriff der Form. Die Konsequenzen würden über allem dämmern. Aber wir halten das nicht ein, wir halten die Grenze ein.

Es gibt eine inhärente Trennung in der universellen Materie, die es ihr ermöglicht, so zu existieren, wie wir sie kennen/wahrnehmen. Wenn es eine Welt ohne Grenzen gibt, sind wir nicht darin aufgetaucht, wir sind hier erschienen.

Nicht gerade eine Antwort, aber mehr gibt es nicht zu sagen

AKTUALISIEREN

Als Antwort auf @Davors Kommentar:

Es ist schwer vorstellbar, was genau passieren würde, weil wir nicht sicher sein können, was die tatsächliche zugrunde liegende Struktur der Realität ist, die die Lichtgeschwindigkeit ausgleicht, und wie sie mit dem Rest der Dinge verflochten ist. Das heißt - wie würden die anderen 3 Kräfte die Materie bilden, wenn EM sofort wäre. Aber lassen Sie uns einige Optionen untersuchen, die den Begriff unterstützen:

  1. Es gibt 4 Kräfte, und wenn die EM-Kraft die gesamte Energie sofort übertragen würde, dann würde sie, selbst wenn die 3 anderen Kräfte noch anhalten würden, die meisten Strukturen, die größer sind als Atome der Materie, die wir jetzt sehen, ungültig machen. Da auf Makroebene nur Schwerkraft und EM von Bedeutung sind und Schwerkraft in diesem Szenario keinen großen Sinn haben wird, werden alle Prozesse, die über EM-Kräfte durchgeführt werden, sofort stattfinden.

  2. Es könnte keine Makrostruktur existieren, und da alle EM-Prozesse unendlich schnell ablaufen, gäbe es keine Möglichkeit, irgendetwas über Elektronenzustände in den Atomen zu wissen. Sie werden unendlich alle möglichen Zustände werden. Alle möglichen Absorptionen und Emission werden gleichzeitig stattfinden. Nicht einmal sicher, ob Atome halten würden.

  3. Versuchen Sie, stattdessen unendlich zu setzen C in allen Beziehungen und sehen, was passiert. Auch, da alle Geschwindigkeiten effektiv als Bruchteil gemessen werden können C , dann wenn C = , sind alle anderen Geschwindigkeiten unabhängig vom Bruchkoeffizienten ebenfalls unendlich.

  4. Siehe die Antwort von @Nikos M.

Kommentare sind nicht für längere Diskussionen gedacht; diese Konversation wurde in den Chat verschoben .
Sie sagen, wenn die Kausalität keine Geschwindigkeitsbegrenzung hätte, würden alle Prozesse mit unendlicher Geschwindigkeit ablaufen?


Das obige Bild habe ich gezeichnet, um Kostyas wunderbare Antwort zu erweitern.

Stellen Sie sich im Grunde Menschen vor, die die Höhe von Gebäuden in Winkelgraden der Sichtbarkeit der Gebäude aus einer bestimmten festen Entfernung messen. Dies ist keineswegs unvernünftig, wenn Sie den Abstand C groß genug im Vergleich zu den Gebäudehöhen festsetzen. Bei höheren Gebäuden werden Sie jedoch feststellen, dass ihre Winkelhöhe nicht addiert wird. Auch die maximal mögliche Winkelhöhe ist auf einen absoluten Wert von 90 Grad festgelegt.

Dies ist der Art und Weise, wie Menschen Geschwindigkeit messen, sehr ähnlich: Wir haben ein bestimmtes Maß „Entfernung/Zeit“ ausgewählt, das für kleinere Geschwindigkeiten sinnvoll ist, aber für höhere Geschwindigkeiten ist es nicht additiv. Außerdem gibt es die unerreichbare "maximale" Geschwindigkeit, die Lichtgeschwindigkeit.

Das obige Problem ist jedoch ausschließlich auf die falsche Wahl der Messgeschwindigkeit zurückzuführen. Die „richtige“ Wahl der Messgeschwindigkeit ist „Schnelligkeit“, wie Kostya erklärt. Und Schnelligkeit ist sowohl additiv als auch unbegrenzt.

Dies ist wahrscheinlich eher ein Kommentar für Kostya, aber diese Antwort ist bereits überfüllt. Es scheint mir, dass diese beiden Antworten ein bisschen kreisförmig sind. Wenn die Nichtadditivität der Geschwindigkeit eine maximale Geschwindigkeit impliziert, lautet die Frage: „Warum ist Geschwindigkeit nicht additiv?“ Soweit ich gesehen habe, beruhen alle Versuche, diese Frage zu beantworten, darauf, dass die Geschwindigkeit der Kausalität begrenzt ist. Damit wird die Argumentation zirkulär.

Wissen wir, WARUM es in unserem Universum eine Geschwindigkeitsbegrenzung gibt?

Deine Frage ist ähnlich wie:

"Wissen wir, WARUM es eine Längenbegrenzung gibt?"

So wie wir endliche Längen brauchen, um die Größe oder den Abstand zwischen zwei Punkten im 3D-euklidischen Raum zu messen , brauchen wir eine endliche Lichtgeschwindigkeit, um den Abstand zwischen Ereignissen im 4D-Minkowski-Raum zu messen . Minkowski entwickelte seine Theorie, um die Maxwell-Gleichung in vier Dimensionen zu erweitern. Damit S 2 im Minkowski-Raum (2), um seine Invarianz zu bewahren, als Erweiterung des Satzes von Pythagoras (1), der in drei Dimensionen lautet:

(1) S 2 = X 2 + j 2 + z 2 ,

und in vier Dimensionen wird:

(2) S 2 = X 2 + j 2 + z 2 ( C T ) 2 ,
C muss nicht nur endlich, sondern für alle Referenzrahmen gleich sein, was durch die beobachteten Beweise gestützt wird, dass die Lichtgeschwindigkeit (EM-Wellen) unabhängig vom Referenzrahmen der Beobachter war.

Die Lichtgeschwindigkeit muss eine Grenze haben, dh endlich sein, damit die Spezielle Relativitätstheorie funktioniert:

Wenn Sie zu den Gedankenexperimenten des berühmten A. Einstein zurückkehren, insbesondere zu dem, bei dem es zwei Beobachter gibt, von denen einer sich nicht bewegt, A , stehend auf einem Bahnhof und einem anderen, B , bewegt sich stehend in einem Zug, der durch den Bahnhof fährt.

Geben Sie hier die Bildbeschreibung ein

Jetzt, wenn der Zug vorbeifährt und A Und B direkt gegenüber liegen, Blitzeinschläge auf beiden Seiten A , im gleichen Abstand. A sieht sie gleichzeitig:

Geben Sie hier die Bildbeschreibung ein

Allerdings, weil B sich relativ zu ihnen bewegt, dh von dem einen weg und auf das andere zu, sieht sie nacheinander:

Geben Sie hier die Bildbeschreibung ein

...Rechts?

Nun, NEIN, dies hätte dazu geführt, dass das Licht in verschiedenen Referenzrahmen unterschiedlich gemessen wurde, was durch das Michelson-Morley-Experiment widerlegt wurde , bei dem die Erde als Zug verwendet wurde :
,

und gemessene Lichtgeschwindigkeit in zwei senkrecht zueinander stehenden Richtungen:

Geben Sie hier die Bildbeschreibung ein

Unter der Annahme, dass das Licht, das sich in die Richtung bewegt, die der Bewegungsrichtung der Erde entspricht, kleiner sein müsste (ähnlich der Person, B , der im Zug saß) als die andere Annahme, die sich wissenschaftlich als falsch erwiesen hat.

Folglich ist die Lichtgeschwindigkeit konstant und jeder, der sie misst, wird den gleichen Wert finden, unabhängig von seiner Geschwindigkeit, oder mit anderen Worten, die Lichtgeschwindigkeit ist unveränderlich . Invarianz könnte der Relativität gegenübergestellt werden, zum Beispiel der Relativität der Zeit, die übrigens verwendet wurde, um zu beschreiben, warum die beiden Menschen, A Und B , beobachten die gleiche Lichtgeschwindigkeit, nämlich wegen der Zeit der sich bewegenden Person B langsamer tickt, gilt im Allgemeinen, je größer die Relativgeschwindigkeit zwischen den beiden Beobachtern ist, desto größer ist der Unterschied in der Tickrate ihrer Uhren, dh die Zeitdilatation .

Schließlich könnte die Zeitdilatation in Gegenwart eines Objekts mit Masse beobachtet werden, das ein Gravitationsfeld erzeugt, oder in Bezug auf die Allgemeine Relativitätstheorie in einer gestreckten Raumzeit, die dazu führt, dass die Zeit des Beobachters, der sich näher am Masseobjekt befindet, tickt langsameren Gang, dh Zeitdilatation bzw. der weiter entfernte Beobachter sieht seine Uhr schneller ticken.

Wie Sie sehen, ist die Zeit relativ, der Raum dehnt sich aus und die Lichtgeschwindigkeit ist die Konstante mit einem endlichen Wert, der sie „zusammenhält“ und „synchronisiert“ , wodurch die Gleichzeitigkeit von Ereignissen definiert wird. Außerdem können wir mit seiner Hilfe ein invariantes Intervall zwischen zwei Punkten in der Raumzeit, also zwischen zwei Ereignissen definieren . Raumzeitintervalle hängen von den zeitlichen und räumlichen Abständen der beiden Punkte ab und können sein: zeitartig , lichtartig (Zeitabstand = Raumabstand) oder raumartig (Zeitabstand < Raumabstand) . Dank dieser speziellen Relativitätstheorieist eine erfolgreiche Theorie mit einer langen Liste unterstützender experimenteller Beweise.

Bearbeiten:

Als Antwort auf den ersten Kommentar, der argumentiert, dass die Zeitdilatation ein Ergebnis der Konstanz der Lichtgeschwindigkeit ist, werde ich Ihnen ein Beispiel für das Gegenteil präsentieren, dh die Konstanz der Lichtgeschwindigkeit kann als direkte Folge der Geschwindigkeits-Zeitdilatation gezeigt werden:

Betrachten wir eine hypothetische Uhr namens Photonenuhr. Darin wird Licht zwischen seinen Spiegeln hin und her reflektiert und immer wenn das Licht auf einen bestimmten Spiegel trifft, tickt die Uhr einmal. Befindet sich diese Uhr in Bezug auf einen Beobachter in einer Trägheitsbewegung, dann wird sie, wie alle anderen Arten von Uhren, durch Geschwindigkeits-Zeit-Dilatation langsamer ticken. Da sich die Uhr jedoch bewegt, verfolgt der Lichtimpuls einen längeren, abgewinkelten Weg zwischen den Spiegeln. Das Nettoergebnis der Geschwindigkeits-Zeit-Dilatation und der Zunahme der Weglänge ist, dass die Lichtgeschwindigkeit in der sich bewegenden Photonenuhr gleich der Lichtgeschwindigkeit in der ruhenden Photonenuhr bleibt. Mit anderen Worten, die Lichtgeschwindigkeit bleibt konstant.

Darüber hinaus sagte die Lorentz-Transformation (LT), die 1897 von Joseph Larmor [1] und Lorentz (1899, 1904) [2] abgeleitet wurde, die Zeitdilatation direkt voraus. Tatsächlich wurde die Zeitdilatation durch den Lorentz-Faktor von Joseph Larmor (1897)[3] richtig vorhergesagt, lange bevor Einstein seine Arbeit 1905 veröffentlichte.


Ihre Frage neigt dazu, ein wenig philosophisch zu sein, soweit wir den Wert kennen C könnte mit einer Eigenschaft der Raumzeit selbst zusammenhängen, die zusammen mit den anderen fundamentalen physikalischen Konstanten während des Urknalls bestimmt wurde, was schwierig zu beobachten und zu realisieren ist, ähnlich wie der Grund, warum Fische nicht das ganze Wasser um sich herum wahrnehmen.

[1] Larmor, J. (1897), „Nach einer dynamischen Theorie des elektrischen und leuchtenden Mediums“, Philosophical Transactions of the Royal Society 190: 205-300.
[2] Lorentz, Hendrik Antoon (1899), „Vereinfachte Theorie elektrischer und optischer Phänomene in bewegten Systemen“, Proc. Akad. Wissenschaft Amsterdam I: 427-443; und Lorentz, Hendrik Antoon (1904), „Elektromagnetische Phänomene in einem System, das sich mit einer beliebigen Geschwindigkeit bewegt, die kleiner als die Lichtgeschwindigkeit ist“, Proc. Akad. Wissenschaft Amsterdam IV: 669-678.
[3] Larmor, J. (1897), „Über eine dynamische Theorie des elektrischen und leuchtenden Mediums, Teil 3, Beziehungen zu materiellen Medien“, Phil. Trans.Roy. Soc. 190: 205-300

Viele der Antworten hier konzentrieren sich auf die falsche Hälfte des Problems, denke ich. Sie sagen Ihnen, woher wir wissen, dass es eine Grenze gibt, anstatt zu erklären, warum es so sein muss.

Größtenteils steht nichts der Erschaffung eines Universums mit unendlicher Lichtgeschwindigkeit entgegen, das dem unseren ansonsten ähnlich ist 1 . Es gibt jedoch eine wichtige Eigenschaft, die ein solches Universum haben müsste: Es muss endlich und/oder inhomogen sein.

Dies hängt mit Olbers Paradoxon zusammen . Im Grunde geht das so:

  1. Das Licht eines Sterns in einer bestimmten Entfernung ist umgekehrt proportional zum Quadrat der Entfernung. L 1 D 2 .
  2. Die Anzahl der Sterne in einer bestimmten Entfernung ist direkt proportional zum Quadrat der Entfernung. N D 2 .
  3. Das Gesamtlicht von Sternen in einer bestimmten Entfernung ist gleich dem Licht pro Stern mal der Anzahl der Sterne. T Licht = L N Licht = L Licht Stern N Sterne . ("Licht" und "Stern(e)" sind hier Einheiten.)
  4. Daher ist das Gesamtlicht aus einer Entfernung gleich dem Licht aus jeder anderen Entfernung. T L N D 2 1 D 2 1 .
  5. Wenn Sterne homogen über das Universum verteilt sind und das Universum unendlich ist, können wir die Sterne in eine unendliche Anzahl von Schalen aufteilen, wobei jede Schale eine endliche, konstante Helligkeit hat. Die Summe all dieses Lichts ist unendlich. D = 0 T = .

Im realen Universum ist dies aufgrund von zwei Effekten kein Problem: Erstens bedeutet die Expansion des Universums, dass sich jeder Stern im Universum (im Durchschnitt) von uns entfernt, und die Rückzugsrate ist direkt proportional zur Entfernung des Sterns ; und zweitens, weil die endliche Lichtgeschwindigkeit bedeutet, dass Licht von fernen Sternen aufgrund dieser Ausdehnung länger braucht, um uns zu erreichen, als es sollte.

Zusammengenommen bedeutet dies, dass das Licht, das uns pro Sekunde von einer bestimmten Sternenhülle erreicht, linear mit der Entfernung abfällt. L 1 D . Dies bedeutet auch, dass es eine endliche Entfernung gibt, in der sich alle Objekte in dieser Entfernung schneller als mit Lichtgeschwindigkeit von uns entfernen, sodass wir niemals Licht von jenseits dieser Entfernung sehen werden (was das Universum effektiv endlich macht). D = 0 N , N < T D < .

Aber wenn Licht mit unendlicher Geschwindigkeit reisen würde, würde Expansion nicht helfen. Es würde bedeuten, dass die Helligkeit des Universums langsam abnimmt (Sterne sind weiter verbreitet, sodass die Helligkeit pro Schale geringer ist), aber eine Abnahme der unendlichen Helligkeit hilft nicht viel. Das Universum müsste also endlich groß sein und/oder die Sterne müssten weniger dicht sein, je weiter man sich vom Zentrum entfernt.

Alternativ könnten wir ein Universum postulieren, in dem das Licht schneller schwächer wird. L 1 D 3 oder so. Aber das macht geometrisch keinen Sinn mehr und erfordert zusätzliche Fummelei, um zu arbeiten. Vielleicht wird das Licht irgendwie vom Expansionsmechanismus absorbiert, wobei das Licht leichter absorbiert wird, da seine Leistung nahe Null ist. Unendlicher Raum bedeutet, dass eine proportional unendliche Lichtabsorption zu einer endlichen Expansionsrate führt und die Lichtdämpfung erhöht. Aber das ist eine Art von Hand-Wavium-Mist, den ich mir ausgedacht habe, und gehört mehr zum Weltenbau als zur aktuellen Physik.

Selbst wenn das Universum endlich wäre, beachten Sie, dass die Helligkeit aller Sterne in unserer eigenen Galaxie die Sonne bei weitem überstrahlen würde. Wir können also wirklich kein Universum haben, das fast identisch mit unserem ist, es sei denn, Licht hat eine endliche Geschwindigkeit.

Beachten Sie auch, dass ich nichts über Quantenmechanik weiß oder wie sich die Lichtgeschwindigkeit auf diesen Wissenschaftszweig auswirkt. Es ist möglich, dass die aktuelle Quantenmechanik nicht mit unendlicher Lichtgeschwindigkeit existieren könnte, was Sie vielleicht erklären könnten bedeutet, dass jedes Universum mit unendlicher Lichtgeschwindigkeit völlig anders ist als unseres. 1 Allerdings betrachte ich „unserem Universum ähnlich“ als jedes Universum mit Teilchenphysik, das die Bildung von Planeten, Sternen, Nervenbahnen usw. auf einer Makroebene ermöglicht, die ein typischer Mensch als ähnlich erkennen würde. Es ist mir egal, ob "Gold" mehr Protonen hat als "Wasserstoff" usw., geschweige denn Quanteneffekte.

Schauen Sie sich die Physik von Greg Egans orthogonaler Reihe an. Ihre Argumente setzen eine feste globale Zeit wie Galileo voraus, was nicht die einzige Lösung ist. Egan hat 4 Dimensionen, wobei die Zeit eine relative Richtung ist: unendliche Geschwindigkeiten (für den Referenzrahmen orthogonaler Beobachter) und keine Olber-Katastrophe.
@JDługosz kein Olbers-Paradoxon, aber daneben ein paar ziemlich seltsame Sachen! Das ist ein großartiger Text - ich bin normalerweise kein Fan von SciFi, aber Orthogonal hat es für mich getan.
Wie könnte ich als Wiederholungstäter und lebenslanger Fan von Olbers' Paradox auf diese Antwort verzichten? (Außerdem sorgt es für Vorhersagbarkeit.) +1!

Ich kann Ihrer Frage entnehmen, dass Sie nach einer einfachen und grundlegenden Erklärung ohne Fachjargon suchen. Ich werde es ehrlich versuchen und es wirklich einfach und klassisch halten. Ich bin ein klassischer Denker, also habe ich nicht einmal eine komplexere Erklärung. Ich hoffe, qualifiziertere und akkreditierte Benutzer werden die Antwort nicht missbilligen.

Lassen Sie mich die Frage in zwei Teile unterteilen -

(1) Warum gibt es eine bestimmte Geschwindigkeit elektromagnetischer Wellen (die zufällig c ist)

Die Geschwindigkeit jeder Welle ist eine Eigenschaft des Mediums, durch das sie sich bewegt. Es ist also eine Eigenschaft des leeren Raums, dass sich elektromagnetische Wellen mit einer bestimmten Geschwindigkeit ausbreiten (nicht mehr und nicht weniger). Es ist eine Eigenschaft, keine Grenze. Wenn es eine Grenze wäre, dann könnte Licht (oder EM) bei < c durch den leeren Raum reisen. Aber es bewegt sich genau bei c, im leeren Raum. Es handelt sich also um eine Immobilie. Wenn Sie mit dieser Erklärung ein Problem haben, sollten Sie auch ein Problem damit haben, dass Schall eine bestimmte Geschwindigkeit durch die Luft hat, und Sie müssen auf eine einfachere Ebene gehen. Wenn Sie eine bestimmte Schallgeschwindigkeit akzeptieren, erwarte ich, dass Sie diese Eigenschaft auch bei EM-Wellen akzeptieren. Der Wert dieser Eigenschaft ist zufällig c. Auch James Large hat dies in seiner Antwort auf Ihre Frage vom 21. Januar angedeutet.

(2) Warum sich kein materieller Körper schneller bewegen kann als c

Dies ist eine direkte Folge von (1) und stellt sich somit als Grenzwert heraus. (Es ist eine Grenze, weil sich Körper mit jeder Geschwindigkeit bewegen können, solange sie c nicht überschreitet.)

Betrachten wir, wie wir die Geschwindigkeit einer Masse erhöhen – wir üben eine Kraft auf sie aus. Zum Beispiel können wir ein stehendes Auto bewegen, indem wir es mit unseren Händen schieben. Die Elektronen in unseren Händen und die im Auto (wo wir es berühren) stoßen sich gegenseitig ab, und diese Abstoßungskraft erhöht die Geschwindigkeit des Autos. Angenommen, Sie würden mit Ihrer Höchstgeschwindigkeit laufen und ein Auto fährt mit 300 Meilen/Stunde an Ihnen vorbei. Kannst du die Geschwindigkeit des Autos erhöhen, indem du mit deinen Händen drückst, wenn es an dir vorbeifährt? Die Antwort ist nein (ein normaler Mensch kann die Hände nicht schneller als 300 Meilen/h bewegen). Um die Geschwindigkeit eines sich bewegenden Körpers zu erhöhen, muss die Kraft schneller auf ihn einwirken als die Geschwindigkeit des sich bewegenden Körpers.

Jede Kraft, die wir anwenden, um einen Körper zu beschleunigen, wird letztendlich als eine der fundamentalen Kräfte auf den Körper ausgeübt. Alle fundamentalen Kräfte selbst bewegen sich bei c. Nur als Beispiel, elektromagnetische Kraft breitet sich mit der gleichen Geschwindigkeit aus wie EM-Wellen, dh der Einfachheit halber einigen wir uns darauf, dass sich alle fundamentalen Kräfte bei c als Eigenschaft gemäß (1) ausbreiten. Daher können sie die Geschwindigkeit eines materiellen Körpers, der sich bereits bei a bewegt, nicht erhöhen Geschwindigkeit von c bis > c.

Beachten Sie, dass die Kräfte schneller als c durch den Raum reisen müssen, um eine Geschwindigkeit größer als c zu verursachen. Aber wir wissen, dass sie sich bei c bewegen. Für einen Körper, der sich bereits bei c bewegt, werden also auch die Grundkräfte wirkungslos.

Die Kräfte werden in Bewegungsrichtung des Körpers bei c unwirksam. In andere Richtungen sind sie noch wirksam, so kann der Körper abgebremst werden etc.

Daher ist die Ausbreitungsgeschwindigkeit fundamentaler Kräfte eine Eigenschaft (keine Grenze). Diese Eigenschaft hat den höchsten Wert im leeren Raum, der zufällig c ist. Und nichts kann sich schneller ausbreiten als die Kräfte selbst – was wird irgendetwas dazu veranlassen? Die Geschwindigkeit der Kräfte (eine Eigenschaft wie die Schallgeschwindigkeit) erweist sich als Geschwindigkeitsgrenze für materielle Körper. Was überhaupt kein Geheimnis ist.

Darf ich auch anmerken, dass sie in Teilchenbeschleunigern die elektrischen/magnetischen Felder verwenden, um die Teilchen zu beschleunigen, und sie offensichtlich nicht schneller als c beschleunigen können.

Bitte LMK, wenn diese Erklärung für Sie funktioniert.

Die Antwort lautet also: "Geschwindigkeit der Kräfte" ist nicht größer als c? Warum nimmt dann die Masse der Teilchen zu, wenn die Geschwindigkeit zunimmt?
Wie würden Sie die Schwerkraft auf ein Photon erklären, das direkt in ein Schwarzes Loch fällt? Wenn das Photon nicht schneller als c geht, liegt das daran, dass die Schwerkraft keine Kraft darauf ausübt (z. B. gewinnt es keine Masse, Energie, wird nicht blau verschoben)?

Diese Frage hat eine sehr kurze Antwort, verwendet jedoch die Annahme, die alle Relativitätstheorien verwenden. dh die Lichtgeschwindigkeit ist für alle Beobachter konstant.

Basierend auf dieser Annahme ist es trivial zu zeigen, dass ein Ereignis horrison bei einer Geschwindigkeit von c beobachtet wird, wenn versucht wird, unendlich zu beschleunigen.

Um zu beantworten, warum diese Annahme gültig ist, müssen Sie sich die Ableitung der Maxwell-Gleichungen ansehen, die zeigen, dass die Ausbreitungsgeschwindigkeit elektromagnetischer Wellen unabhängig vom Bezugssystem ist. Die Ableitung dieser Gleichungen basiert auf Konzepten wie "Ladungserhaltung" und dem Faradayschen Gesetz. Ich glaube nicht, dass die Frage tiefergehend beantwortet werden kann.

tldr: Durch Energieerhaltung => Ladung und empirische Beobachtungen elektromagnetischer Wechselwirkungen kann man zeigen, dass die Lichtgeschwindigkeit unabhängig vom Inertialsystem ist. Dies kann verwendet werden, um zu beweisen, dass es eine universelle Geschwindigkeitsbegrenzung von c gibt.

Wie Sie aus anderen Antworten gelesen haben, ist es nicht einfach zu erklären. Es scheint so kontraintuitiv. "Wenn ich schneller fahren will, warum beschleunige ich dann nicht einfach mehr?" oder "Wenn meine Geschwindigkeit sehr nahe an der Lichtgeschwindigkeit liegt, dann schieße ich eine Kugel ab, wird sie nicht schneller als Licht sein? Warum nicht?"

Relativität

Beginnen wir mit der Präzisierung dessen, was wir unter „Geschwindigkeitsbegrenzung“ verstehen. Nehmen wir an, Sie befinden sich in einem Raumschiff mit einer unendlichen Treibstoffquelle und einem willkürlichen Beschleunigungspotential. Sie sind in Ruhe (an einer Raumstation angedockt) und möchten zu einem 10 Lichtjahre entfernten Sternensystem reisen. Wie lange wirst du brauchen? In diesem hypothetischen Raumschiff könnten Sie in 10 MINUTEN dorthin gelangen, nicht in Jahren (vorausgesetzt, die Beschleunigung bringt Ihren empfindlichen menschlichen Körper nicht durcheinander).

Aber ist das nicht ein Verstoß gegen die kosmische Geschwindigkeitsbegrenzung? Nö! Technisch gesehen reist du nicht schneller als das Licht. Aus Ihrer Sicht scheint es, als würde der Weltraum flacher und Ihr Ziel nähert sich Ihnen. Wenn jemand auf der Raumstation Sie beim Einsteigen beobachten würde, würden Sie aus seiner Sicht sehr nahe an der Lichtgeschwindigkeit reisen, aber er würde Sie erst in knapp 10 Jahren in der Zukunft an Ihrem Ziel ankommen sehen.

Gehen wir jetzt noch einen Schritt weiter und stellen uns vor, Sie seien ein Lichtstrahl, der durch den Weltraum reist. Wie lange dauert es aus Ihrer Sicht, bis Sie mit etwas interagieren? Überhaupt keine Zeit. Ein Lichtstrahl teleportiert sich sofort von der Quelle zu einem Ziel, ohne dass Zeit vergeht. Aber natürlich gelten hier die gleichen Relativitätsprinzipien - ein außenstehender Beobachter wird diese sofortige Teleportation nicht erleben.

Nicht-Relativität

Stellen Sie sich nun ein alternatives Universum vor, in dem es keine kosmische Geschwindigkeitsbegrenzung gibt. Erstens würde Licht sofort reisen. Wenn wir also in den Himmel blicken, würden wir andere Sterne und Planeten genau so sehen, wie sie gerade sind. Wir könnten in beliebig kurzer Zeit zu und von jedem Ort im Universum reisen. Scheint realistisch oder?

Das Problem ist, was im kleineren Maßstab passiert. Stellen Sie sich einen atomaren Prozess – wie den in unserer Sonne – in diesem hypothetischen Universum vor. Der Kern der Sonne hat eine Temperatur von etwa 15 Millionen Grad Celsius (denken Sie daran, dass die Temperatur mit der kinetischen Energie zusammenhängt). Die Sonne hat einen Durchmesser von etwa 4,6 Lichtsekunden.

Eine relativistische Lichtgeschwindigkeit wirkt als Drossel und verhindert, dass diese atomaren Kettenreaktionen zu schnell ablaufen. Es hilft zu begrenzen, wie heiß etwas sein kann (indem die Masse von Partikeln erhöht wird, die sich sehr schnell bewegen, um zu verhindern, dass sie sich zu schnell bewegen) und wie schnell Reaktionen stattfinden können (Bruchteile einer Sekunde für die Energie, um sich zu bewegen, vs. augenblicklich). ein riesiger Unterschied). Dies könnte bedeuten, dass Sterne zu schnell explodieren, um sich überhaupt zu bilden. Es könnte auch bedeuten, dass die Energie, die die Reaktionen antreibt, der Sonne zu schnell entweicht und ihr keine Zeit zum Reagieren gibt. Ich bin mir nicht sicher, was passieren würde, aber so oder so sind die Ergebnisse katastrophal.

Abschluss

Das „kosmische Tempolimit“ ist ein wichtiger Aspekt unseres Universums. Es ist zwar denkbar, ein Universum ohne relative Geschwindigkeitsbegrenzung zu entwerfen, aber die Ergebnisse wären nicht sehr interessant. Unsere Intuition sagt uns, dass dies möglich sein sollte, aber manchmal ist unsere Intuition bei Dingen, die wir nicht vollständig verstehen, nicht sehr gut.

Die Frage "Warum gibt es eine kosmische Geschwindigkeitsbegrenzung?" ist so grundlegend wie "Warum enthält das Universum mehr Materie als Antimaterie?" oder "Warum existiert Magnetismus?". Die Frage sollte neu formuliert werden: "Warum leben wir in einem Universum mit diesen Eigenschaften?" Oder "Könnten wir in einem Universum mit anderen Eigenschaften leben?" Weil es möglich ist, dass andere Universen mit anderen Eigenschaften existieren, und nur ein kleiner Teil davon kann tatsächlich Leben erhalten. Wenn menschliches Leben existiert, wird es dies natürlich in einem Universum tun, das es erhalten kann.

Dieser Beitrag untersucht einige der Folgen eines Tempolimits, geht aber nicht wirklich auf das Warum ein, außer dem „klassischen“ muss es so sein, sonst gäbe es uns nicht. In gewisser Weise ist das eine so gute Antwort wie jede andere, nehme ich an.
Es ist, als würde man fragen: "Warum ist die Erde 93 Millionen Meilen von der Sonne entfernt?" Die Antwort ist, niemand weiß es genau. Es hat sich einfach so ergeben. Die Frage selbst ist fehlerhaft - sie ist völlig irrelevant. Die eigentliche Frage ist: "Könnten wir überhaupt existieren, um die Frage zu stellen, ob die Erde in einer anderen Entfernung wäre?"
Sie sagten: "Wenn jemand von der Raumstation Sie beim Einsteigen beobachten würde, würden Sie aus seiner Sicht sehr nahe an der Lichtgeschwindigkeit reisen" und sie würden Sie erst 10 Minuten vor Ihrer Ankunft "sehen", richtig? Sonst ist es absurd. Ich sehe kein Licht „einschiffen“, bis es ankommt.
@nocomprende Ich bin mir nicht sicher, was du meinst. Ich spreche nicht vom leichten Einschiffen, ich spreche vom Schiff. Jeder Beobachter würde sehen, wie das Schiff etwa 10 Jahre braucht, um das Ziel zu erreichen, aber die Person auf dem Schiff würde nur 10 Minuten vergehen sehen.
„Wir würden andere Sterne und Planeten genau so sehen, wie sie gerade sind. Wir könnten ohne Zeitverlust zu und von jedem Ort im Universum reisen.“ - Was? wie impliziert ersteres letzteres?
@immibis: Technisch gesehen könntest du mit unendlicher Geschwindigkeit reisen, wenn du dich in Daten umwandelst und die Daten mit Lichtgeschwindigkeit sendest. Aber Sie würden Energie benötigen, die proportional zum Quadrat der Entfernung ist, damit der Empfänger das Signal tatsächlich verstehen und Sie rekonstruieren kann, also wären es keine unendlichen Entfernungen.
@MIchaelS Wenn das als "Reisen" gilt, ist das umstritten. Es ist jedoch definitiv nicht "bewegt".
@immibis du nimmst diesen Absatz zu wörtlich. Ich versuche, eine intuitive Erklärung dafür zu vermitteln, wie ein Universum ohne Geschwindigkeitsbegrenzung aussehen würde. Wenn es keine Geschwindigkeitsbegrenzung gibt, könnten Sie beliebig schnell reisen, was bedeutet, dass Sie jede Entfernung in einer beliebig kurzen Zeit zurücklegen können.
Ein Proton bei 15 Millionen Kelvin ist nicht relativistisch. Es scheint keinen Grund dafür zu geben, dass die Sonne explodiert, wenn wir die Idee der speziellen Relativitätstheorie aufgeben und Aktionen aus der Ferne zulassen.
@LLlAMnYP Berechnungen auf der Rückseite des Umschlags legen nahe, dass sich Atome mit etwa 0,2% der Lichtgeschwindigkeit in der Sonne bewegen, mit einer Zeitdilatation von etwa 0,0002%. Und natürlich würde sich elektromagnetische Strahlung sofort ausbreiten. Die Dinge wären sicherlich anders, aber ich stimme zu, dass eine Explosion möglicherweise nicht das Ergebnis ist.
Die Lichtgeschwindigkeit und die Unfähigkeit, die Lichtgeschwindigkeit zu überschreiten, sind nicht unbedingt ein und dasselbe. Warum konnte die Lichtgeschwindigkeit nicht so sein, wie sie ist, aber man konnte in einem Raumschiff beliebig schneller reisen? Nehmen Sie als Beispiel die Schallgeschwindigkeit und unsere Fähigkeit, sie zu übertreffen.
@Neil Sie sind dasselbe. Wir sollten es nicht einmal "Lichtgeschwindigkeit" nennen, wir sollten es wirklich "relative Geschwindigkeitsbegrenzung" nennen, und eine Eigenschaft des Lichts ist, dass es sich mit dieser Geschwindigkeit fortbewegt. Und wieder können Sie schneller als dieses Limit reisen, aber nur von einem bestimmten Standpunkt aus.
@JamesWatkins Das ist ein anthropisches Argument. Das wäre so, als würden Sie auf die Frage, warum der Himmel blau ist, antworten: "Weil es nur so sein kann." Das mag in diesem Fall sogar die richtige Antwort sein, aber Sie verfehlen den Punkt. Vielleicht ist die bessere Frage hier: " Warum können sie keine getrennten Konzepte sein?"
@Neil Das könnten sie sein. Es gibt keine Regel, die besagt: "Du kannst kein Universum ohne Geschwindigkeitsbegrenzung haben." Die allgemeine Frage lautet also: "Warum hat unser Universum einen?" Und das ist so grundlegend wie die Frage: "Warum hat unser Universum Magnetismus?" Außerhalb schwerer Mathematik gibt es keine befriedigende Erklärung.

Im Gegensatz zu den anderen Antworten werde ich versuchen, eine einfache Antwort zu geben.

Seien Sie sich zunächst bewusst, dass „Warum“ eine schlechte Frage für die moderne Wissenschaft ist, da die moderne Wissenschaft es vorzieht, „was“ passieren wird, so genau wie möglich vorherzusagen, indem sie „Modelle“ dessen verwendet, was ihrer Meinung nach die Realität tut.

Geschwindigkeit und Zeit sind stark miteinander verbunden und unterliegen effektiv derselben "Geschwindigkeitsbegrenzung". Wenn du so langsam wie möglich durch den Raum gehst, dann gehst du so schnell wie möglich durch die Zeit und umgekehrt.

Unser beobachtbares Universum scheint eine universelle "Raum-Zeit"-Grenze zu haben. Diese Grenze ist Teil der miteinander verbundenen „ Fundamentalkonstanten “ unseres Universums.

Das Interessante ist, dass, wenn einer von ihnen in irgendeiner Weise signifikant verändert würde, unsere aktuellen Modelle sehr unterschiedliche Versionen der Realität vorhersagen, in denen es sehr unwahrscheinlich ist, dass empfindungsfähiges Leben entstehen würde (bitte entschuldigen Sie die wilden Spekulationen hier), um dies fragen zu können Frage.

Dies nennt man das anthropische Prinzip

Also die Antwort auf deine Frage

Wissen wir, WARUM es in unserem Universum eine Geschwindigkeitsbegrenzung gibt?

ist "ja, weil wir das Glück haben, in einem Universum zu leben, in dem scheinbar zufällige Geschwindigkeitsbegrenzungen empfindungsfähiges Leben entstehen ließen"

Es mag viele solcher Universen geben, möglicherweise mit anderen fundamentalen Konstanten als unserem, möglicherweise mit radikal unterschiedlichen Formen von empfindungsfähigem Leben.

Die wahren PHYSIK-Fragen hinter all dieser Philosophie beinhalten

  • "Gibt es andere Universen?"
  • "Was können wir über sie wissen?"
  • "Wie sind die Wahrscheinlichkeitsverteilungen der Fundamentalkonstanten in diesen Universen?"
  • usw usw usw

Leider glaube ich nicht, dass praktische wissenschaftliche Experimente vorgeschlagen wurden, um diese Modelle zu testen.

Daher ist dieses Thema eher Philosophie als Physik , also sollte Ihre Frage wahrscheinlich als nicht zum Thema gehörend geschlossen werden?

Wortlaut als Antwort auf Kommentar aktualisiert.

FYI das anthropische Prinzip ist Spekulation. Es wird als zusätzlicher Faktor verwendet, um den Zoo zu reduzieren, in den sich die Stringtheorie ohne gute Lösung begeben hat. Wenn das Universum anders wäre, wäre das Leben anders, einfach so, kein Grund, sich glücklicher als nötig zu fühlen. Beachten Sie den Unterschied zwischen verschiedenen Lebensformen und keinem Leben
Einige sagen, das anthropische Prinzip sei einfach ausgedrückt das Folgende: "Das Universum ist so und so, damit Kosmologen es beobachten können". Nicht gut für meine Verhältnisse
Das anthropische Prinzip besagt nicht „das ist das einzig mögliche Universum, in dem sich Leben entwickeln könnte“. Es heißt einfach: "Wenn dieses Universum kein Leben entwickeln könnte, wäre niemand hier, um zu fragen, warum nicht". Es ist ein Argument gegen "das Universum wurde speziell für uns entworfen", nicht gegen die Existenz alternativer Universen.
"Wenn du so langsam wie möglich durch den Weltraum gehst, dann gehst du so schnell wie möglich durch die Zeit und umgekehrt." Gibt es eine bekannte Gleichung, die diese Beziehung modelliert?
@JamesWatkins Ja, die Lorentz-Gleichung beschreibt die Zeitdilatation im Kontext der speziellen Relativitätstheorie. siehe simple.wikipedia.org/wiki/Time_dilation

Nun, es ist möglich zu beweisen (theoretisch, und ich rate Ihnen für Feigenbaum, 2008 ), dass die Homogenität und Isotropie des Raums und die Homogenität der Zeit notwendigerweise zur Existenz einer Geschwindigkeitsbegrenzung führen.

Machen wir das: Stellen Sie sich vor, Sie nehmen das Universum und löschen (entfernen) alle Arten von Objekten. Du bleibst nur bei der Raumzeit selbst. In dieser Raumzeit gibt es nichts, egal keine Energie.

Nun: ist es vernünftig zu glauben, dass es in einer leeren Raumzeit einen einzigen Punkt gibt, der privilegierter ist als der andere? Nein, also ist die leere Raumzeit homogen.

Ist es vernünftig zu glauben, dass in einer leeren Raumzeit eine Richtung privilegiert ist gegenüber den anderen? Nein, also ist die Raumzeit auch isotrop.

Aus diesen beiden Annahmen zeigt Feigenbaum die Existenz einer Grenzgeschwindigkeit. Andererseits ist es auch interessant festzustellen, dass in Einsteins Theorie die Existenz einer Grenzgeschwindigkeit ein Axiom ist. Dies ist jedoch in gewissem Sinne unnötig. In der Tat kann man unter Annahme weniger Dinge (wie nur Homogenität und Isotropie) zeigen, dass es eine Grenzgeschwindigkeit geben muss.

Nun, die Tatsache, dass DIESE Grenzgeschwindigkeit die Lichtgeschwindigkeit ist, ist eine Frage, die Feigenbaums Theorie weder zeigt noch beweist. Diese Tatsache muss durch ein Experiment behoben werden!

Letzter Vorbehalt

Was bleibt wirklich wahr in einer Raumzeit voller Materie? Alles, was ich oben geschrieben habe, läuft immer noch und es ist gültig, aber nur lokal , nämlich in wirklich kleinen Bereichen der Raumzeit und Bereich für Bereich.

Global gesehen hat es keine Bedeutung zu sagen, dass es eine Grenzgeschwindigkeit gibt (weil das Konzept einer globalen Geschwindigkeit selbst schlecht definiert ist) und es kann vorkommen, dass sich manchmal mit einer überlichten Geschwindigkeit bewegt. Zum Beispiel: Weit entfernte Galaxien bewegen sich weit weg von uns mit Geschwindigkeiten, die weit über der Lichtgeschwindigkeit liegen.

Vielen Dank für Ihre Antwort, aber wie sind Sie vom isotropen und homogenen Universum zu einer Geschwindigkeitsbegrenzung gekommen? (Ich bin Ihrer Argumentation nicht vollständig gefolgt)
@LandosAdam diese Art folgt aus der galiläischen Relativitätstheorie. Sobald Sie fordern, dass die Physik in allen Referenzrahmen gleich ist, kommen Sie schnell zu der Forderung, dass beispielsweise die Lichtgeschwindigkeit in allen Referenzrahmen gleich ist, und das führt zu Lorenz-Invarianz und SR. Aber was wäre, wenn es tatsächlich einen privilegierten Bezugsrahmen gäbe? Nicht viele Dinge bewegen sich relativ zu uns mit relativistischer Geschwindigkeit, daher sehe ich nicht ein, warum ein bevorzugter Referenzrahmen viel Material zerstören sollte.
wäre gut, den tatsächlichen Verweis auf den Feigenbaum-Artikel bereitzustellen. Beachten Sie, dass Homogenität und Isotropie verwendet werden können, um eine Geschwindigkeitsbegrenzung abzuleiten, aber in dem Sinne nicht grundlegend sind, dass Raumzeit mit absolut keiner Materie und Energie isotrop und homogen sein kann, aber nicht viel bedeutet. Andererseits gilt MIT Materie und Energie nicht zwangsläufig Isotropie und Homogenität. Man kann die Geschwindigkeitsbegrenzung auch aus dem Relativitätsprinzip ableiten (Achtung, nicht aus den Relativitätstheorien ), das wiederum wiederum aus der Kausalität abgeleitet werden kann
Ich habe den Link zu Feigenbaums Artikel über arxiv hinzugefügt. Allerdings sollte die Antwort auch den entscheidenden Teil der mathematischen Herleitung aus dem Paper enthalten, ansonsten handelt es sich um eine reine Link-Antwort.
Vielleicht möchten Sie einen Blick auf arxiv.org/abs/1209.0563 und die Einstein Digital Papers hier und hier werfen . Der Raum ist nicht homogen und die Lichtgeschwindigkeit variiert.

Man kann sagen, dass dies nur ein experimentelles Ergebnis ist. Dieses Licht (und andere Signale / Wechselwirkungen) reisen nicht mit unendlicher Geschwindigkeit / Geschwindigkeit.

Man kann es dabei belassen und sagen, dass es so ist.

Man kann auch sagen, schauen Sie, wenn Sie diese Variable nehmen und diese Transformation (z. B. rapidity ) durchführen, kann sie ad infinitum hinzugefügt werden, also dreht sich die Frage um die richtige zu verwendende Variable. Obwohl dies nur an der eigentlichen Frage vorbeigeht, anstatt sie anzusprechen.

Ich werde einen anderen Ansatz wählen und die Frage direkt ans Herz richten.

Die endliche Geschwindigkeit der Signalübertragung (oder Wechselwirkung) ist eine Grundvoraussetzung für die Gültigkeit der Kausalität .

Andernfalls, wenn die Signalübertragung unendlich sein kann, kann eine Wirkung ihre eigene Ursache überholen , und die resultierenden Kausalschleifen lassen die Kausalität aufhören, eine Kausalität zu sein. Etwas, das ebenfalls eine experimentelle Tatsache ist, aber auf einer noch grundlegenderen Ebene. In diesem Sinne ist dies die Antwort auf die Frage.

Man kann dies noch weiter gehen und eine (obere) endliche Geschwindigkeitsübertragung direkt aus thermodynamischen Überlegungen ableiten, die außerhalb des Bereichs dieser Frage liegen, aber für weitere Untersuchungen erwähnt werden.

Die endliche Geschwindigkeit der Signalübertragung entspricht jedoch nicht unbedingt dem Postulat der Speziellen Relativitätstheorie , dass die Lichtgeschwindigkeit diese maximal (und konstante) verfügbare Geschwindigkeit ist.

Abhängig vom untersuchten Prozess kann man viele verschiedene endliche Übertragungsgeschwindigkeiten haben, die kleiner oder sogar größer als die Lichtgeschwindigkeit sind.

Tatsächlich gibt es einige Untersuchungen zur Signalübertragung schneller als Licht durch Quantenverschränkung . Aber das lasse ich an dieser Stelle einfach

Aber Kausalität könnte der Effekt sein, dass es eine Obergrenze für die Geschwindigkeit gibt, mit der Informationen übertragen werden können, wie Sie sagten. Sie können es also nicht als Argument verwenden, um zu erklären, warum die Grenze überhaupt existiert, weil Sie im Nachhinein sprechen
@LandosAdam, hmm, man sollte als grundlegend nehmen, was einfacher erscheint. Kausalität ist ein tieferes und erweitertes Konzept, das über den Bereich der Mechanik und Geschwindigkeit hinausgeht. In diesem Sinne wird es als grundlegender angesehen. Aber ich kann abschweifen, wenn das Gegenteil mit der gleichen Allgemeinheit gegeben ist :)
Aber ich spreche nur von Kausalität für die Dinge, die in unserem Universum aufgrund der Grenzgeschwindigkeit passieren, nicht von Kausalität im allgemeinen Sinne. Oder anders ausgedrückt: In einem alternativen Universum, in dem es keine Obergrenze gäbe, gäbe es keine Kausalität für Dinge wie die, die Sie erwähnt haben. (Kausalität würde jedoch in ihrer allgemeinen Form existieren)
@LandosAdam, hmm, kannst du das klären? ich denke du wirst es nicht schaffen. In jedem Fall, wenn gezeigt wird, dass die beiden äquivalent sind (oder sagen wir isomorphe Konzepte ), dann gibt es kein Problem, welches zu verwenden ist. Wenn nicht, dann ist das eine allgemeiner als das andere. Bisher scheint es Kausalität zu sein
In diesem alternativen Universum hätte Kausalität mit Signalen (aus Ihrem eigenen Beispiel) nicht einmal eine Bedeutung, eine Existenz. Es würde ihnen einfach nichts ausmachen, wenn die Wirkung ihrer eigenen Ursache vorauseilt, weil ihr Universum und die Art und Weise, wie es funktioniert, grundlegend anders wären. Ich finde..! Ich kann es nicht mehr erklären, da ich mir nicht einmal sicher bin, was ich sage, und ich im Moment (mit all den Annahmen) antiwissenschaftlich sein könnte! Auf jeden Fall verstehe ich Ihren Standpunkt, aber ich habe nur das Gefühl, dass die Kausalität, von der Sie sprechen, ein Produkt der Geschwindigkeitsbegrenzung ist und sicherlich nicht umgekehrt
Ja, genau das verstehe ich auch. Die meisten Strukturen und Interaktionen, die uns umgeben, werden durch EM geformt. Wenn EM unendlich schnell wird, wird all dies unendlich schnell. Und idk, ob der Rest der Kräfte mit der Sache mithalten wird. Ist Zeit ... elektromagnetisch?
@LandosAdam, ich verstehe, was würde als Antwort darauf zählen, warum endliche Geschwindigkeitsbegrenzung? Haben Sie eine Idee oder einen Vorschlag? es wäre schön zu hören. Beachten Sie, dass in der Antwort erwähnt wird, dass die Thermodynamik ohne andere Annahmen eine endliche Geschwindigkeitssignalübertragung ableiten kann (vielleicht ist dies aus dieser Perspektive besser?).
Nein, ich habe keine Ahnung, ich versuche nur, die Ideen der Experten herauszufinden (weil ich noch kein Experte bin), aber ich versuche, dies zu tun, ohne mich selbst zu täuschen, indem ich jedem Argument „glaube“, das man vorbringen könnte . Es scheint mir nur, dass viele der Dinge, an die die meisten Leute denken, um meine Frage zu beantworten, auf der Existenz der Grenze beruhen, anstatt zu erklären, warum sie existiert (wenn es ein Warum gibt). Tut mir leid, wenn ich kritisch werde, aber ich möchte nur konstruktiv argumentieren.
@noncom, ich bin mir nicht sicher, ob Zeit (oder Dauer ?;)) elektromagnetisch ist, aber die meisten Wechselwirkungen (vorerst ohne Schwerkraft) sind tatsächlich Verallgemeinerungen des Elektromagnetismus (auch bekannt als Yang-Mills-Theorie).
@LandosAdam, nein, überhaupt kein Problem, schließlich ist dies Ihre Frage. Siehe eine ähnliche Frage von mir hier (sagen wir mal in die gleiche Richtung). Ich habe selbst eine Antwort, aber ich wollte hören, was andere sagen könnten
Angenommen, das Universum folgte der galiläischen Relativitätstheorie. Auf welche Weise würde dann die Kausalität verletzt? Es folgt bereits bei niedrigen Geschwindigkeiten ungefähr der Galileischen Relativitätstheorie, aber wir verletzen die Kausalität bei niedrigen Geschwindigkeiten nicht.
Wenn Informationen augenblicklich zwischen Objekten in beliebiger Entfernung übertragen werden könnten, wie würde das die Kausalität aufheben? Wenn wir nicht in der Lage wären, die Reihenfolge von Ursache und Wirkung zu bestimmen, weil zwischen den beiden keine schöne Dauerlücke besteht, würde das die Kausalität stoppen oder nur unsere Wahrnehmung der Kausalität stören?
@Corey: Kausalität wird nicht verletzt. Die Leute sind einfach zu sehr an SR-Gleichungen hängen, die solche Verletzungen vorhersagen, basierend auf dem, was wir derzeit über unser Universum wissen. Informationen wären immer noch nicht sofort verfügbar. Physische Strukturen würden aus massiven Teilchen bestehen, die endliche Geschwindigkeiten hätten, so dass es immer eine gewisse Verzögerung zwischen Übertragung, Empfang, Verständnis, Reaktion usw. geben würde. Außerdem könnte die Wirkung die Ursache niemals in einem Moment erreichen, bevor die Ursache eintritt. also ist die Kausalität immer noch nicht verletzt.
@MichaelS Es verwirrt mich nur, wenn Leute behaupten, dass die Kausalität unter bestimmten Bedingungen zusammenbricht, die anscheinend nicht zu einem Konflikt mit der Kausalität führen. Vielleicht denken sie so über die Zeit?
@immibis, das Argument ist umstritten, da die galileische Relativität oder nicht die endliche Signalübertragung nicht ausschließt. Sie verwechseln endliche Signalübertragung mit maximaler und konstanter Geschwindigkeit der speziellen Relativitätstheorie. Im Sinne dieses Beitrags ist die spezielle Relativitätstheorie genau das, ein Sonderfall. Nicht mehr und nicht weniger. Tatsächlich können durch die Thermodynamik endliche Signalgeschwindigkeiten (oder charakteristische Geschwindigkeiten ohne Annahme der speziellen Relativitätstheorie abgeleitet werden).
@Corey, siehe meinen vorherigen Kommentar. Die Unterscheidung zwischen Kausalität und Kausalitätswahrnehmung ist einfach deshalb nicht gut ausgedrückt, weil, wenn etwas existiert, man es irgendwie erfahren kann (nicht unbedingt auf eine bestimmte Weise). Wenn es absolut keine Möglichkeit gibt, Kausalität (oder irgendetwas anderes) zu erfahren, dann ist es ein und dasselbe, ob sie existiert oder nicht, nichts und bedeutungslos. Punkt klar, ich hoffe, das verwirrt Sie nicht
@MichaelS, siehe meine vorherigen Kommentare, hoffe, sie beantworten alle Einwände, die Sie haben könnten
@NikosM. Es ist eine Analogie. Wenn die galiläische Relativitätstheorie gelten würde, würde eine endliche Lichtgeschwindigkeit die Kausalität immer noch nicht verletzen, genauso wenig wie eine endliche Geschwindigkeit von Gummibällen die Kausalität verletzt. Es würde nur bedeuten, dass Licht nicht das Schnellstmögliche war.
Jeder hier spielt immer wieder auf SR an, wenn ihm effektiv die Frage gestellt wird: "Was ist, wenn die galiläische Relativitätstheorie gilt?" Aber was, wenn wir einen absoluten Bezugsrahmen haben müssen und die Lichtgeschwindigkeit nicht für alle Beobachter gleich ist? Würde dann etwas grundlegend im Universum zusammenbrechen? Würden sich nicht Sterne und Leben bilden? Es gibt viele Prozesse, für deren Beschreibung kein SR oder GR erforderlich ist.
@LLlAMnYP ja alles würde kaputt gehen. Elektromagnetismus und Licht würden Äquivalente benötigen, die auf völlig andere Weise bereitgestellt werden. Ähnliche Phänomene auf Makroebene zu haben, würde eine völlig andere Implementierung auf niedriger Ebene erfordern. Auch die Details der schwachen Kraft, die Sterne zum Leuchten bringt, ergeben sich aus den beteiligten Symmetrien auf der untersten Ebene. Das Ändern der Raumzeit-Symmetriegruppe zieht den Teppich unter ihr weg. Denken Sie an den Versuch, x86-Maschinencode auf einem ARM-Prozessor auszuführen!

Es ist nicht besonders ungewöhnlich, dass physikalische Systeme Geschwindigkeitsbegrenzungen haben.

Betrachten Sie die klassische vibrierende elastische Saite, definiert durch die Gleichung

2 T 2 j ( X , T ) = A 2 X 2 j ( X , T )

Mit dieser Gleichung können Sie sehen, dass sich eine kleine Störung in einem Teil der Saite mit einer bestimmten Geschwindigkeit nach außen ausbreitet. Tatsächlich werden Sie sehen, dass die Geschwindigkeit, mit der sich eine Störung entlang der Saite ausbreiten kann, durch diese Geschwindigkeit begrenzt ist.

Sie können sich diese Geschwindigkeit intuitiv vorstellen, indem Sie sich vorstellen, dass die Schnur aus kleinen Perlen besteht, die durch elastische Fäden verbunden sind, und das Signal sich von Perle zu Perle ausbreiten muss, was seine Geschwindigkeit begrenzt.

Sie werden ein ähnliches Phänomen in den Differentialgleichungen für einen elastischen 3D-Festkörper (wie einen Wackelpuddingwürfel) sehen. Auch die Differentialgleichungen für ein elektrisches Signal in einem Draht oder eine elektromagnetische Welle im Raum oder eine Schallwelle in der Luft.

So ziemlich jedes System, das durch eine Differentialgleichung beschrieben werden kann, die die Änderungsrate im Laufe der Zeit mit einer lokalen Eigenschaft wie der Ableitung oder der Dichte in Beziehung setzt, hat am Ende eine Geschwindigkeitsbegrenzung. Und es ist ziemlich üblich, dass sich physikalische Systeme so verhalten, weil die meisten Dinge auf der Welt aus kleineren Teilen bestehen und das makroskopische Verhalten des Systems anhand des Verhaltens der kleineren Teile analysiert werden kann.

Mir ist klar, dass dies überhaupt nicht beantwortet, warum . Ich möchte nur darauf hinweisen, dass es keine so ungewöhnliche oder überraschende Sache ist, ein Tempolimit zu haben.

Einfach gesagt, es ist der Weg der Natur, die Kausalität zu bewahren . Aus Wikipedia :

„Wenn sich Signale andererseits schneller als mit Lichtgeschwindigkeit bewegen könnten, würde dies die Kausalität verletzen, weil es ermöglichen würde, dass ein Signal über raumähnliche Intervalle gesendet wird, was bedeutet, dass das Signal zumindest für einige Trägheitsbeobachter zeitlich rückwärts reisen würde . Aus diesem Grund erlaubt die spezielle Relativitätstheorie keine Kommunikation schneller als mit Lichtgeschwindigkeit.“

Wenn es keine Geschwindigkeitsbegrenzung gäbe, wäre jede Art von Paradoxon mit Verletzung der Kausalität möglich (vielleicht haben Sie vom Großvater-Paradoxon gehört ?).

Aber diese Antwort hat, wie die meisten anderen, mit der Wirkung der Einschränkung zu tun und nicht mit dem Grund dahinter.
Dann ist eine Antwort glaube ich nicht möglich. Das ist wie die Frage: "Warum ist Kraft gleich Masse mal Beschleunigung?" oder "Warum wird die Quantenmechanik von der Schrödinger-Gleichung bestimmt?" ...
Nun, ich dachte nicht wirklich, dass es eine Antwort geben würde. Ich habe nur gefragt, für den Fall, dass es eine Antwort gibt, weil ich gerade in meinen ersten Schritten als Physiker bin, also gibt es eine Menge Dinge, die ich nicht weiß und nicht ableiten kann (wenn sie ableitbar sind)! :)
Warum ist Ihre Antwort so niedrig? Du hast 100% Recht! Wir brauchen eine Geschwindigkeitsbegrenzung (und nicht nur eine Begrenzung, sondern eine unveränderliche Begrenzung), damit die Kausalität in unseren Relativitätstheorien unter Druck gesetzt werden kann ... Für Galileo war es unsere Geschwindigkeit durch die Zeit, sie war konstant, unveränderlich und gleich 1 s/s. ... Für Einstein ist es unsere Geschwindigkeit durch die Raumzeit, sie ist konstant, unveränderlich und gleich 1 ls/s ... Wir brauchen nur eine Geschwindigkeitsbegrenzung, um die Kausalität zu wahren ... Warum müssen wir das tun? Weil wir ANNAHMEN, dass die Welt deterministisch ist (zumindest in großen Maßstäben), bauen wir eine deterministische Theorie auf, um die Welt so zu erklären, wie wir sie verstehen
"Ist unsere Welt deterministisch?" ist die Folgefrage, und bisher ist sie eine philosophische, daher kann sie hier nicht beantwortet werden. Der "Grund" hinter der Begrenzung ist die Wahrung der Kausalität ... Niemand zu Galileos Zeiten hat jemals gefragt: "Warum muss ich jede Sekunde 1 Sekunde altern? Kann ich nicht schneller oder langsamer altern?" ... Nein, weil das so war nicht unintuitiv, also hat das niemand in Frage gestellt... Niemand bis Einstein, der das verstanden hat JA! Sie können tatsächlich schneller oder langsamer altern! Aber in der Raumzeit MUSS man immer mit 1ls/s gehen, sonst bricht man die Kausalität! Oder zumindest werden unsere Theorien nicht in der Lage sein, sie zu verfolgen!

Die Frage in der Überschrift lautete

Wissen wir, warum es in unserem Universum ein Tempolimit gibt?“

Dann gab es eine Verstärkung

Bei dieser Frage geht es darum, warum wir eine universelle Geschwindigkeitsbegrenzung haben (die Lichtgeschwindigkeit im Vakuum). Gibt es ein grundlegenderes Gesetz, das uns sagt, warum das so ist? Ich frage nicht, warum das Tempolimit gleich c ist und nicht etwas anderes, sondern warum es überhaupt ein Limit gibt.

Ich denke, dass @Anna_v von allen Antworten der Beantwortung der Frage am nächsten gekommen ist.

Derzeit lautet die Antwort auf die Frage „Nein“.

Die Idee , dass es eine universelle Geschwindigkeitsbegrenzung gibt, stammt von Beobachtungen des Universums.
Diese Beobachtungen führen zu einigen Theorien, die verwendet werden können, um Vorhersagen über das Universum zu treffen.
Einige dieser Theorien, die gute Vorhersagen treffen können, gehen davon aus, dass es eine universelle Geschwindigkeitsbegrenzung gibt.
Anders gesagt, das universelle Tempolimit ist ein nützliches Postulat, weil es einige der Theorien „funktionieren“ lässt.
Gegenwärtige Theorien sind nicht in der Lage, alles vorherzusagen, was passiert ist und passieren wird, und daher suchen Wissenschaftler nach besseren Theorien.

Ob es eine Theorie gibt, die erklärt, warum es eine universelle Geschwindigkeitsbegrenzung gibt, ist unbekannt.

Derzeit existiert eine solche Theorie nicht.

Eine physikalische Antwort:

Wenn sich ein elektrisch geladener Körper relativ zu einem Beobachter bewegt, könnte ein induziertes Magnetfeld vom Beobachter gemessen werden. Die in diesem Magnetfeld gespeicherte Energie geht gegen unendlich, während sich die Geschwindigkeit des sich bewegenden Körpers c nähert.

Dies ist mit der Allgemeinen Relativitätstheorie nicht ganz intuitiv ... Sagen Sie auch, dass zwei geladene Körper, die bei 0,9 c aufeinander zusteuern, für einen stationären Beobachter an dem Punkt, an dem sie sich treffen, ein größeres induziertes Magnetfeld haben, als jeder Reisende beobachtet? (Ist es nur 2x für den externen Beobachter, oder spielt es verrückt, weil die Deltageschwindigkeit zwischen den beiden sich nähernden Objekten 1,8c beträgt?)
@BenPen, alles hängt von der Bewegung des Beobachters ab. Und das Magnetfeld ist in verschiedenen Trägheitssystemen unterschiedlich, obwohl alle Systeme "im selben Universum untergebracht" sind.

Mir ist aufgefallen, dass die Gleichungen umso mehr wie Koordinatentransformationen aussehen, je näher man grundlegenden physikalischen Theorien kommt, denjenigen, die die grundlegendsten Wechselwirkungen in unserem Universum beschreiben. Manchmal befinden sich diese Koordinaten in abstrakten Räumen – den Gruppen des Standardmodells der Teilchenphysik und den Hilbert-Räumen der Quantenmechanik – aber letztendlich ist die Physik eine Beschreibung der Bewegung von Dingen.

Um etwas im Universum zu lokalisieren, braucht man Ort und Zeit. Nun, auch wenn Sie beim Lesen still sitzen, bewegen Sie sich durch die Zeit. Die Geschwindigkeit, mit der Sie sich durch die Zeit bewegen, beträgt laut Ihrer eigenen Uhr eine Sekunde pro Sekunde, aber nicht jeder wird damit einverstanden sein. Lassen Sie uns herausfinden, wie schnell Sie nach Meinung eines Beobachters fahren.

Lassen Sie sich mit einem großen Bruchteil der Lichtgeschwindigkeit auf eine Reise zum Alpha Centauri-Sternensystem schicken. Ein Bewohner an Ihrem Zielort beobachtet Ihre Fahrt und sieht, dass Sie eine Strecke von zurückgelegt haben D (etwa 4 Lichtjahre). Laut der Uhr, die Sie getragen haben, sind Sie um eine Zeit gealtert T , was weniger ist als die Zeit, die der Alpha Centaurian für Ihre Reise aufgrund von Zeitdilatation gemessen hat. Um Ihre gesamte Reise durch die Raumzeit zu finden, können wir Ihre Reise in zwei Dimensionen mit dem Satz des Pythagoras kombinieren:

X = D 2 + T 2 .
Die Gesamtstrecke, die Sie in Raum und Zeit zurückgelegt haben, ist X ; die Entfernung, die Sie im Weltraum zurückgelegt haben, ist D ; und die Entfernung, die Sie in der Zeit zurückgelegt haben, ist T (was gleichbedeutend mit der Angabe ist, wie viel Sie gealtert haben). Das Problem mit dieser Gleichung ist das D Und T haben unterschiedliche Einheiten: Meter und Sekunden. Glücklicherweise liefert Einsteins Relativitätstheorie einen Umrechnungsfaktor: die Lichtgeschwindigkeit. Die Gleichung sollte also lauten:
X = D 2 + ( C T ) 2 .

Nun ist die zurückgelegte Entfernung gleich der Geschwindigkeit des Raumschiffs multipliziert mit der Zeit der Reise, gemessen vom Alpha Centurion (Die Entfernung, die im Ruhesystem von zwei Punkten gemessen wird, wird als Eigenentfernung bezeichnet. Die Eigenzeit wird von einer ruhenden Uhr mit gemessen die zeitgesteuerte Entität, nämlich Ihre Uhr.). Nennen wir die verstrichene Zeit in Alpha Centauri T a .

X = ( v T a ) 2 + ( C T ) 2 .
Wir können uns beziehen T Und T a mit der Zeitdilatationsgleichung:
T = T a γ = T a 1 ( v C ) 2
Wo γ ist der relativistische Faktor, der in fast allen relativistischen Gleichungen vorkommt. Beachte das T a ist kleiner als T um die langsamere Alterung widerzuspiegeln, der sich schnell bewegende Objekte (Sie) unterziehen.

Also, jetzt haben wir

X = ( v T a ) 2 + ( C T a 1 ( v C ) 2 ) 2 .
Vereinfachung:
X = v 2 T a 2 + C 2 T a 2 ( 1 ( v C ) 2 ) = T a v 2 + C 2 ( 1 ( v C ) 2 ) = T a v 2 + C 2 v 2 = T a C 2 = C T a .
Die Gesamtstrecke, die Sie durch Raum und Zeit zurückgelegt haben, entspricht der Lichtgeschwindigkeit mal Ihrer Reisezeit. Dies gilt unabhängig von Ihrer Geschwindigkeit. Wenn Sie sich also gemeinsam durch Raum und Zeit bewegen , bewegen Sie sich immer mit Lichtgeschwindigkeit! Zu denken, dass sich verschiedene Objekte mit unterschiedlichen Geschwindigkeiten fortbewegen, ignoriert ihre Bewegung durch die Zeit. Die Lichtgeschwindigkeit ist also nicht nur eine Höchstgeschwindigkeit. Es ist auch eine Mindestgeschwindigkeit. Man könnte sagen, es ist die einzige Geschwindigkeit.

Eine Folge davon ist, dass man sich umso langsamer durch die Zeit bewegt, je schneller man sich durch den Raum bewegt und umgekehrt. Sie können sich diese Situation so vorstellen, als würden Sie in einem Auto ohne Gaspedal und ohne Bremspedal fahren – nur mit einem Lenkrad. Es fährt immer mit der gleichen Geschwindigkeit. Wer nach Osten fahren möchte, muss in nördlicher Richtung etwas Geschwindigkeit opfern. Genauso muss man, wenn man sich durch den Raum bewegen will, etwas Geschwindigkeit durch die Zeit opfern. Tatsächlich funktioniert die Mathematik für die Relativitätstheorie genauso, wenn Sie sich eine Achse als Raum und die andere als Zeit vorstellen, wie ich es in der obigen Ableitung getan habe.

Gibt es nun einen grundsätzlichen Grund zu beantworten, warum das so ist? Das Beste, was mir einfällt, ist zu beobachten, dass wir keine Kontrolle darüber haben, wie schnell wir altern. Ein Jahr für Sie ist genau das gleiche Jahr für mich (es sei denn, es werden bald fortschrittliche Raumfahrzeuge erfunden). Wenn sich die Zeit nicht so sehr vom Raum unterscheidet, wie es in unserem Universum scheint, dann wäre auch die Reise durch den Raum, wie die Zeit, auf eine bestimmte Geschwindigkeit begrenzt. Die grundlegende Tatsache über unser Universum, die die Lichtgeschwindigkeit begrenzt, ist die gegenseitige Abhängigkeit von Bewegung durch Raum und Zeit.

Du bist viel zu niedrig für eine so elegante Antwort :/

Warum gibt es in unserem Universum ein Tempolimit? Das könnte etwas mit dem Ortsprinzip in der Physik zu tun haben. Beachten Sie, dass es in Fredkins Universum als zellulärem Automaten immer eine Geschwindigkeitsbegrenzung für jedes aufkommende Muster gibt (nur ein Beispiel). Die Existenz einer Geschwindigkeitsbegrenzung in unserem Universum ist also eine Bestätigung (Folge) des Prinzips der Lokalität in der Physik. Als Randbemerkung müssen Quanten-Nichtlokalitätsargumente (basierend auf Quantenverschränkungsexperimenten) in Bezug auf synchronisierte chaotische Systeme erklärt werden, ohne das Prinzip der Lokalität abzulehnen.

Eine gewisse Konstante c mit Dimensionen einer Geschwindigkeit ist notwendig, da Boosts nicht pendeln und daher Boosts durch dimensionsloses (mathematisches) Radiant erfolgen müssen. Die Konstante c wandelt Geschwindigkeiten in Radiant um. Die Konstante c kann nicht unendlich sein, da dies dazu führen würde, dass Boosts pendeln.

Einem Objekt eine Geschwindigkeit (Boosten) in der x-Richtung zu geben, pendelt nicht mit dem Boosten in der y-Richtung. Boosts und Rotationen gehorchen empirisch der Definition einer Gruppe. Die spezielle Relativitätstheorie entdeckte, dass Boosts Mitglieder der nicht-abelschen Lorentz-Gruppe sind. Für v C 1 es stimmt, dass

Schub ( v X C ) Schub ( v j C ) Schub ( v j C ) Schub ( v X C ) = Drehung z ( v X C v j C ) .

Es muss eine Konstante geben C mit Geschwindigkeitsdimensionen, um die Boost-Parameter dimensionslos im Bogenmaß zu machen, so dass ihr Produkt (der Rotationswinkel um die z -Achse) kann auch im dimensionslosen Bogenmaß angegeben werden. Das ist in Ordnung R A D ich A N S 2 = R A D ich A N S wie durch die Terme in der Potenzreihenentwicklung von belegt Sünde ( θ ) . Es ist Unsinn, darüber eine Rotation zu machen z -Achse durch ( M S ) 2 .

Wenn C , dann würden die Boosts pendeln und sie wären nicht mehr Teil der Lorentz-Gruppe. Die Konstante c ist ähnlich der Konstante A = ( 180 π ) D e G R e e S die verwendet wird, um Winkel umzuwandeln Θ von Grad in Radiant. Die Rotationsgruppe (die eine Untergruppe der Lorentz-Gruppe ist) ist nicht abelsch

Drehung ( Θ X A ) Drehung ( Θ j A ) Drehung ( Θ j A ) Drehung ( Θ X A ) = Drehung z ( Θ X A Θ j A ) .

Der " A " ist notwendig. Es wäre Unsinn, eine Rotation darüber zu machen z -Achse durch [ Grad ] 2 . Wenn A , dann würden die Rotationen pendeln und sie wären nicht mehr Teil der Lorentz-Gruppe. Wenn Rotationen pendeln würden, würde unsere Welt ganz anders aussehen. Es würde nicht so etwas geben, ein Objekt um einen Winkel ungleich Null zu drehen und es in seine ursprüngliche Ausrichtung zurückkehren zu lassen. Außerdem wäre der Drehimpuls nicht quantisiert und Teilchen hätten keinen Spin.

Zusammenfassend sind c (und a) notwendig und müssen endlich sein, da Boosts (und Rotationen) Teil der nicht-abelschen Lorentz-Gruppe sind. In dieser Gruppe befindet sich der Boost-Parameter [ Schnelligkeit ] = Tanh 1 ( v C ) kommt aus Kostyas Antwort.

Nette Herangehensweise an das Thema, aber es setzt die Schlussfolgerung, bevor es implizit erreicht wird. Die Annahme von Raumzeitsymmetrien zur Erfüllung der Lorentz-Gruppe läuft auf die Konstanz der Lichtgeschwindigkeit hinaus ...
Wir sind an den historischen Weg zu SR gewöhnt, bei dem zuerst die raumzeitliche Symmetrieinvarianz der Minkowski-Metrik entdeckt wurde und ac benötigt wurde, um ct die gleiche Dimension wie x zu geben. Dass diese Symmetrietransformationen die Lorentz-Gruppe bilden, erfuhren wir als nächstes als Ableitung von dieser Symmetrie.
... Nehmen wir jedoch an, wir haben zuerst empirisch herausgefunden, dass das Produkt von Boosts nicht nur Geschwindigkeiten hinzufügt, sondern eine gewisse Rotation verursacht und dass Boosts nicht pendeln. Indem wir sehen, wie sich Produkte verhalten, stellen wir fest, dass wir die Gruppenelemente von O(3,1) mit Rotationen und Boosts identifizieren und unsere Symbole auf einem Blatt Papier erfolgreich dazu bringen können, die Beobachtungen der realen Welt zu kopieren. Erst dann sehen wir das Linienelement D S 2 = D X 2 + D j 2 + D z 2 ( C D T ) 2 wird von der Gruppe invariant gelassen. Nun könnten wir sagen, dass die Gruppe der grundlegende Grund dafür ist, dass ac benötigt wird. Die Symmetrieinvarianz der Metrik ist dann das Nebenergebnis.
Die Gruppe von was? ... Von räumlichen und/oder zeitlichen Transformationen, die physikalische Systeme isomorph (in Bezug auf die Physik) auf physikalische Systeme abbilden. Das meinte ich mit Raumzeitsymmetrien. Mein Ausgangspunkt ist nun, dass die Annahme (oder Beobachtung) dieser Struktur direkt die Konstanz der Lichtgeschwindigkeit impliziert. Die Frage nach der Geschwindigkeitsbegrenzung lautet nun, warum diese besondere Gruppenstruktur von der Realität gezeigt wird.

OK, wir sprechen über Geschwindigkeiten und die Tatsache, dass es eine Geschwindigkeitsbegrenzung gibt. Wir sprechen also von Bewegung. Bewegung enthält zwei Variablen. Das eine ist die Geschwindigkeit, das andere die Distanz. Die Variablen reichen von null bis unendlich. Um also das größtmögliche Bild in Bezug auf Bewegung zu betrachten, würde man diese beiden Variablen natürlich ins Unendliche verschieben.

Mit unendlicher "Geschwindigkeit" zu reisen bedeutet, jede Entfernung in Nullzeit zu überwinden. Das heißt, man könnte in sagen wir 1 Minute von Punkt A nach Punkt B reisen, aber das bedeutet auch, dass man schneller reisen und die Fahrt in weniger als sagen wir nur 1 Sekunde beenden könnte. Je schneller Sie fahren, desto weniger Zeit wird benötigt. Das sind endliche Geschwindigkeiten. Aber wenn Sie in kürzester Zeit von A nach B reisen, dann ist diese besondere Geschwindigkeit nicht zu überbieten. Das ist die unendliche Geschwindigkeit.

Nächste. Über eine unendliche "Entfernung" zu reisen bedeutet, dass Sie für immer weitergehen werden, da eine unendliche Entfernung kein Ende hat. Wenn Sie also die beiden kombinieren und mit unendlicher Geschwindigkeit über eine unendliche Entfernung reisen, bedeutet dies, dass Sie in kürzester Zeit für immer weitermachen werden . Ganzheitlich ist dies einfach nicht möglich. Relativistisch gesehen ist es jedoch möglich, und es ist möglich, da die beiden Extreme getrennt werden.

In einem möglichen Extrem können Sie sich durch den Raum bewegen, aber nicht durch die Zeit. Im anderen möglichen Extrem bewegen Sie sich durch die Zeit, aber nicht durch den Raum. Um diesen Übergang zu ermöglichen, müssen Sie, wenn Sie in einem gewinnen, in dem anderen verlieren. Sie können also nicht beide Extreme gleichzeitig haben. Daher müssen sie ihrerseits in eine Endlichkeit eingeschlossen sein, um diese Gewinn- und Verlustphänomene hervorzurufen.

Im Gegenzug bewegst du dich vielleicht durch den Raum, aber nicht durch die Zeit, während für diejenigen, die dich beobachten, die Zeit immer noch tickt, und daher, obwohl die Zeit für dich stillsteht, die Zeit woanders ewig weitergehen kann. So können Sie in kürzester Zeit ewig weitermachen.

Somit ist eine endliche Bewegung über die Raumzeit eine Voraussetzung, um Bewegung zu ermöglichen. Somit gibt es eine endliche Grenze der Bewegungsgeschwindigkeit im Raum.

Wenn Sie dann das Ergebnis dieses Phänomens analysieren, entdecken Sie unabhängig die Spezielle Relativitätstheorie und leiten unabhängig alle ihre mathematischen Gleichungen ab. Sehen Sie sich dieses Video zur Überprüfung an.

Bitte geben Sie bei Ablehnung eine Erklärung ab. Schließlich hatte mich mein logisches Denken bezüglich einer einfachen Analyse von "Bewegung" zu meiner unabhängigen Entdeckung der SR-Phänomene und zu meiner unabhängigen Ableitung der SR-Gleichungen geführt, und ich tat dies auf eine Weise, auf die sonst niemand gekommen ist für jetzt. Wenn meine Logik falsch ist, dann gibt es keine Geschwindigkeitsbegrenzung, und die SR-Gleichungen sind auch falsch. Zugegeben, ich habe überhaupt keine vorherige Physikausbildung erhalten. Aber ich habe gelernt, dass man als rückständig gilt, wenn man SR alleine entdeckt. Dies kann sich auf eventuell erscheinende negative Voten beziehen.
Ich bin nicht der Downvoter, aber ich denke, die allgemeine Idee ist, dass Ihre Erklärung verschwommen ist (nicht streng genug, um Ansprüche abzuleiten). Hier ist der Grund: Zu sagen, dass „[wenn Sie] mit unendlicher Geschwindigkeit über eine unendliche Entfernung reisen, bedeutet dies, dass Sie in kürzester Zeit für immer weitermachen werden.“ ist falsch. Sie löschen Unendlichkeiten, ohne das asymptotische Verhalten Ihrer Mengen anzugeben. Es läuft darauf hinaus, zu sagen 0 = , was undefiniert ist. Zweitens unterstellen Sie implizit die Schlussfolgerung, wenn Sie Ihre beiden Extreme nennen, indem Sie die Zeit bereits einer räumlichen Dimension ähneln.
G. Bergeron - Nun, Sie könnten sich immer meine YT-Videos ansehen und sehen, was das Ergebnis ist. Sie sind über mein Netzwerkprofil zugänglich.

Ausgangspunkt für Einsteins Argumentation war, dass Raumzeitkoordinaten kein physikalisches Vorhergehendes sind, sondern aus physikalischen Messverfahren ermittelt werden. Nehmen wir das allgemeine Relativitätsprinzip an

  • Lokale Gesetze der Physik sind die gleichen, unabhängig von der Referenzmaterie, die ein bestimmter Beobachter verwendet, um sie zu quantifizieren

dann richten alle Beobachter die Koordinaten auf die gleiche Weise ein, was bedeutet, dass eine maximale Geschwindigkeit (falls vorhanden) für alle Beobachter gleich ist. Entweder es gibt in der Natur eine Höchstgeschwindigkeit oder nicht. Abgesehen von dem Argument, dass das Fehlen einer maximalen Geschwindigkeit der Beobachtung widerspricht, können wir feststellen, dass alle physikalischen Prozesse Zeit brauchen. Das Fehlen einer Höchstgeschwindigkeit würde die Möglichkeit einer sofortigen Aktion in einer Entfernung (zumindest in einer Grenze) implizieren, die Newton als beschrieben hat

eine so große Absurdität, dass ich glaube, dass kein Mann, der in philosophischen Angelegenheiten eine kompetente Denkfähigkeit hat, jemals darauf hereinfallen kann .

In GR ist die Lichtgeschwindigkeit nicht konstant, sie variiert mit der Krümmung der Raumzeit. Die Konstanz dieser universellen Geschwindigkeit hängt also von der Raum-Zeit ab, die eine konstante Krümmung hat. Was nicht der Fall ist, aber dies ist lokal eine nützliche Annäherung, und um der Absicht des OP gerecht zu werden, werden wir von nun an annehmen, dass das Universum ein Raum mit konstanter Krümmung ist. In jedem Fall wissen wir, dass die Lichtgeschwindigkeit langsamer ist, wenn die Krümmung größer ist. Wenn wir also nach der Grenze suchen, müssen wir den Fall einer konstanten Krümmung von Null betrachten, da sie sonst überall langsamer ist.

Nehmen wir nun der Einfachheit halber an, dass diese Krümmung Null ist.

Es wurde experimentell beobachtet, dass Masse gleich Energie ist, also haben sie die gleichen Einheiten. Aber die zusätzliche Masse, die durch die kinetische Energie einer Geschwindigkeit v erzeugt wird, ist 1 2 M v 2 also muss v dimensionslos sein. Daher gibt es ein Koordinatensystem für Raum-Zeit, in dem die x-Koordinaten die gleichen Einheiten wie die t-Koordinaten haben. Da die Mannigfaltigkeit völlig flach, praktisch euklidisch ist (bis auf die -1 in der Signatur der Metrik), können wir ein Koordinatensystem wählen, das naiverweise überall gleich ist. Eine raumartige Richtung kann also überall gleichförmig in eine zeitartige Richtung gedreht werden. (Das mag wie SR klingen, ist aber noch nicht SR. Dies ist einfach eine Dimensionsanalyse plus einfache Geometrie plus die eine experimentelle Tatsache der Äquivalenz von Masse und Energie). Aber dann haben wir eine universelle Geschwindigkeit, dieselbe Umwandlung zwischen der x-Koordinate und der t-Koordinate.

Bisher heißt das nicht, dass die Geschwindigkeit ein Tempolimit ist, noch dass es mit Licht zu tun hat. Aber es ist kanonisch und intrinsisch und "physikalisch", da es vom Umwandlungsverhältnis zwischen Masse und Energie abhängt.

Der nächste Schritt besteht darin, abzuleiten, dass dies eine universelle Geschwindigkeitsbegrenzung ist. Das geschieht wie üblich, da durch Beschleunigung die Masse des Objekts zunimmt und somit ein exakt quantitativer "abnehmender Ertrag" gilt.

Alles, was wir wollen, folgt also aus Newtons Beziehung zwischen Masse, kinetischer Energie und Geschwindigkeit, plus der einen experimentellen Tatsache der Masse-Energie-Äquivalenz.

Anmerkung: William Davidson hat irgendwo eine Anmerkung veröffentlicht, die zeigt, wie die gesamte SR aus der Masse-Energie-Äquivalenz folgte. Ich habe es nicht gelesen, aber nur die Tatsache zu hören, dass er es getan hat, hat mich offensichtlich darauf hingewiesen. Also muss man diese "Priorität" anerkennen.

Es kann keinen sehr grundlegenden philosophischen Grund geben, warum Masse mit Energie äquivalent sein muss, da theoretische Physik auf galiläische Weise möglich ist, wo sie nicht wahr ist. Andererseits könnte man den galiläischen Fall philosophisch immer in dem Sinne als in diesen Rahmen eingeschlossen betrachten ist eine universelle Konstante und auch eine universelle Geschwindigkeitsbegrenzung, gleichberechtigt mit 1. (Es ist Null als universelle Geschwindigkeitsbegrenzung, die in der Physik niemals akzeptiert werden könnte ... nicht einmal in der Theorie.)

Bitte nicht zweimal dieselbe Antwort posten zweimal , sondern stimmen Sie dafür ab, doppelte Fragen zu schließen, anstatt Ihre Antwort zu kopieren.
Lieber Josef F. Johnson. Es ist oft verpönt, nahezu identische Antworten zu posten auf ähnliche BeiträgeIn solchen Fällen ist es oft besser, doppelte Fragen einfach zu markieren/kommentieren, damit sie geschlossen werden können.

Die Menge an Masse, die ein Objekt hat = wie widerstandsfähig dieses Objekt gegenüber Impulsänderungen ist. Je mehr Masse, desto mehr Energie wird benötigt, um den Impuls zu ändern. Da Photonen masselos sind, haben sie keinen Widerstand gegen Impulsänderungen. Photonen sind im Wesentlichen die Geschwindigkeit, mit der sich Energie durch das Universum ausbreitet. Warum es c ist, so funktioniert das Universum. Es gibt kein Gesetz, dass sich etwas nicht schneller bewegen kann, aber wenn wir die physische Welt als die Dualität von Energie/Masse betrachten, dann kann sich nichts Physisches schneller bewegen als Licht.

Der Grund, warum es in unserem Universum eine Geschwindigkeitsbegrenzung gibt, ist, dass wir nichts (keine Energie/Kraft) haben, das uns helfen kann, etwas schneller zu bewegen als die höchste uns zur Verfügung stehende Geschwindigkeit ( C ) .

Mit anderen Worten, wenn die Geschwindigkeit von C war 2 10 8 M S (oder 4 10 8 M S ), dann wäre dies die Geschwindigkeitsbegrenzung des Universums. Das Beste, was wir tun könnten, wäre, die Geschwindigkeit von zu erreichen C .

Licht bewegt sich aufgrund seiner Geschwindigkeit, aber auch wegen seines Weges am schnellsten im Universum. Pierre de Fermat behauptete, dass „Licht sich zwischen zwei Punkten auf dem Weg der kürzesten Zeit bewegt“. Dies kann als das Prinzip der kürzesten Zeit bezeichnet werden. Dann gibt es noch das Prinzip der kleinsten Wirkung. Zitat von Pierre Louis Maupertuis "Die Natur ist sparsam in all ihren Handlungen". Können Sie sich ein Beispiel in der Natur vorstellen, wo Ressourcen/Energie verschwendet werden? Ich würde vorsichtig behaupten, dass die Natur keine Bewegung benötigt, die schneller als Licht ist, und sie daher nicht bereitgestellt hat.

Eine Ablehnung bedeutet, dass die Antwort richtig ist (in diesem Fall)

Die Lichtgeschwindigkeit gilt für mehr als nur Licht, nichts kann sich schneller ausbreiten, einschließlich Kräfte, die auf Materie einwirken. Es gibt jedoch eine nicht traditionelle Antwort darauf, warum. Die Lichtgeschwindigkeit ist eine Höchstgeschwindigkeit, weil wir uns alle in einem wellentragenden Medium, der Raumzeit, bewegen. Licht bewegt sich als Welle im Medium mit konstanter Geschwindigkeit darin; und wie alles andere bei der sanften Fortbewegung in einer "Flüssigkeit" bewegen wir uns langsamer als eine Welle, die sich in der Flüssigkeit der Raumzeit fortbewegt, wie es bei Objekten in Wasser oder Luft oder Wellen in Festkörpern beobachtet wird. (Festkörper sind ein Sonderfall, da sie nur von Wellen durchdrungen werden.)

Die Analogie deutet darauf hin, dass es möglicherweise keine absolute Geschwindigkeitsbegrenzung gibt, aber alles, was wir beobachtet haben, reist durch die „Flüssigkeit“ der Raumzeit und kann nicht verwendet werden, um über die Lichtgeschwindigkeit hinaus zu beschleunigen, also stecken wir fest. Es gibt klassischerweise zwei Fälle für die Beschleunigung.

1) Das beschleunigte Ding hat Masse:

Wir haben beobachtet, dass die Masse, wie sie von einem externen Beobachter beobachtet wird, im Verhältnis von variiert

1 1 v 2 / C 2
Unsere Beobachtung besagt also, dass die Masse ins Unendliche geht, wenn v sich c nähert, aber wir wissen nicht genau, „warum“, aber es wirkt wie ein zunehmender Widerstand gegen die Reise durch ein „Medium“ oder eine abnehmende Rückkehr, die versucht, Schwung zu verleihen reisendes Objekt. In Abhängigkeit von der benötigten Energie erhöht sich die Geschwindigkeit, Ihr Ertrag an Geschwindigkeitserhöhung wird immer geringer, weil der Widerstand gegen Bewegung, "Masse", immer größer wird und schließlich, wegen "Masse", Energie (das sind Wellen oder Materie auf dem gleichen Medium wie das Objekt, das wir beschleunigen, also muss es sich langsamer als die Lichtgeschwindigkeit bewegen) wird schließlich so ineffektiv, dass es für keine bekannte Energiequelle wirklich einen Effekt verursacht.

2) Das beschleunigte Ding hat keine Masse:

Das einzige, was wir derzeit kennen, das keine Masse hat, aber auf irgendeine Weise messbar ist, ist ein Wellen-/Teilchen-Ding, und es kann nicht beschleunigen, es bewegt sich einfach durch die Flüssigkeit der Raumzeit. Gravitation, EM wie Licht, fundamentale Kräfte breiten sich alle als Welle auf einem Medium aus. Und noch einmal, dieses Medium ist „Raumzeit“, aber wir wissen nicht, woraus es besteht. Andere Wellen wandern durch Materie, und bei allen gibt es eine messbare Wellengeschwindigkeit. ABER es ist eine Reise in Materie, innerhalb der Raumzeit.

3) Der dritte Fall der Beschleunigung von Materie über die Lichtgeschwindigkeit hinaus ist eine interessante Spekulation, aber letztendlich vergeblich, da dies nach der bekannten Physik nicht möglich ist, da es keine Möglichkeit gibt, mehr Impuls hinzuzufügen, nachdem Sie die Lichtgeschwindigkeit erreicht haben. Wenn jemand eine Kraft hinzufügen würde, die auf etwas anderes als "Raum ..." drückte, gäbe es ein Gedankenexperiment für eine schnellere Reise als Wellen im Medium der Raumzeit.

Wenn Sie sich das schnellere Reisen in anderen Medien ansehen, können Sie sehen, dass es möglich ist, aber die Dinge ändern sich. Eine Zeit lang dachten wir, dass wir nicht schneller als Schall reisen könnten, aber da es keine Beschleunigungsgrenze gibt, passiert etwas Besonderes an der Grenze, ein Flugzeug kann sie überschreiten und es verursacht eine konische Schockwelle dahinter. Ein Boot kann schneller fahren als die Wellen im Wasser, und wieder erscheint eine brechende Welle, eine "Stoßwelle" auf der Oberfläche hinter dem Boot. Was würde es bedeuten zu beschleunigen, um schneller zu reisen als die Wellen, die in der Raumzeit reisen? Wir glauben nicht, dass dies möglich ist, aber es scheint, als wüssten wir, wie es bei relativistischer FTW-Reise mit langsamerer Geschwindigkeit aussehen könnte. Es gibt einen Namen für die Stoßwelle, die entsteht, wenn sich Materie schneller als die lokale Lichtgeschwindigkeit bewegt (ein Spaltreaktor ist das übliche Beispiel). sogenannte Tscherenkow-Strahlung, die mit normaler Geschwindigkeit schneller als Wellenbewegungen in einer Flüssigkeit verfolgt. (Wir sehen sichtbares Licht, einen hübschen Blauton.) Bei Fahrten in normalen Flüssigkeiten führt eine Fahrt, die schneller als eine Welle ist, zu einem neuen Nachlaufregime. Könnte es mit der Lichtgeschwindigkeit, der Geschwindigkeit einer Welle in der Raumzeit, dasselbe sein? Vielleicht, wenn es eine andere Kraft gäbe, eine andere Möglichkeit, dem System Schwung zu verleihen. Etwas, das sich schneller als c bewegt, würde möglicherweise eine Stoßwelle bilden und die Raumzeit möglicherweise auf neue Weise deformieren. Es gibt Hinweise darauf, dass die Raumzeit wie eine durch Masse komprimierbare Flüssigkeit wirkt; Wenn Sie die Schwerkraft als Kraft ausschließen, die direkt auf Licht einwirken könnte, sieht Licht, das sich um eine ausreichend große Masse krümmt, so aus, als würde sich die Geschwindigkeit eines Objekts in der Richtung senkrecht zu seiner Bewegung ändern, was wir Brechung nennen. Es ist wahrscheinlich ein falscher Vergleich, Da es jedoch keine Beweise dafür gibt, dass Licht beschleunigen KANN; sonst würde der Ereignishorizont um ein Schwarzes Loch kein Licht schlucken. Aber es gibt Leute, die zuschauenDichte des Raums Also, es gibt Forschung an den Rändern, aber das Überschreiten dieser Grenze von c ist nicht so wahrscheinlich, aber interessant zu betrachten.

Ich bin anderer Meinung in dem Punkt, dass es in unserem Universum eine universelle Geschwindigkeitsbegrenzung gibt, es sei denn, Sie definieren das Wort "Geschwindigkeit" auf sehr unintuitive Weise.

Ich behaupte, dass die richtige Länge der im Beobachterrahmen zurückgelegten Strecke durch die richtige Zeit im Objektrahmen geteilt wird ( = γ v ) kommt unserem intuitiven Verständnis von Geschwindigkeit näher als die Variable v (was in Bezug auf die nicht intuitive Definition der Zeit in der speziellen Relativitätstheorie definiert ist). Diese Menge ist nicht begrenzt durch C .

Entsprechend kann man die Größe des Vektors nehmen, der nur aus den räumlichen Komponenten der Vierergeschwindigkeit besteht. Auch dies ist nicht gleich v und kann größer sein als C .

Ich bin verwirrt: Das Betragsquadrat des räumlichen Teils des Geschwindigkeitsvektors ist 4 γ 2 v 2 = v 2 / ( 1 v 2 / C 2 ) . Wenn v > C Wie Sie vorschlagen, wird diese Größe negativ, was für das Quadrat der Größe dieses Vektors keinen Sinn ergibt.
Welche Beweise haben Sie dafür D v / D τ ist unbegrenzt?
@ZeroTheHero Es ist nicht v, das größer als c wird, sondern Gamma mal v.
(Was Sie ganz einfach selbst überprüfen können @KyleKanos .)
@MrFrety: Ich bin mir nicht sicher, was du meinst, wie kann ich das selbst überprüfen?
@KyleKanos Warte eine Sekunde, du sprichst von einer Beschleunigung. Ich nehme an, du wolltest x statt v schreiben. Um es etwas ausführlicher zu machen: ZeroTheHero hat das Quadrat der Größe, von der ich spreche, korrekt aufgeschrieben. Versuchen Sie - wie ich es getan habe - einige Zahlen, die sehr nahe an 1 liegen, für v in zu setzen γ v beim Setzen von c auf 1. Sie werden feststellen, dass dieser Ausdruck beliebig weit über 1(=c) hinausgehen kann.
@MrFrety: Hoppla, ja. hatte ich gemeint X . Entschuldigung für die Verwirrung :(
+2 Ja, das ist eine offensichtliche Idee, wenn man anfängt, über die Relativität nachzudenken. Schade, dass diese Seite unkonventionelles Denken nicht toleriert, daher werden die meisten kreativen Antworten stark abgelehnt. Weiter so, guter Kampf. Sie haben jetzt den Ruf, überall zu kommentieren.

Die Tachyon-Theorie könnte einige Erkenntnisse zu dieser Frage bringen. Gemäß der Tachyon-Theorie ist es möglich, dass sich ein Teilchen schneller als die Lichtgeschwindigkeit fortbewegen kann, solange das Teilchen mit einer höheren Geschwindigkeit als der Lichtgeschwindigkeit erzeugt wurde. Und die Lichtgeschwindigkeitsgrenze kann laut Tachyon-Theorie von unten oder oben angefahren, aber nicht überschritten werden. Ein Tachyon mit unendlicher Geschwindigkeit hat keine relativistische Energie , gewinnt aber an Energie, wenn es auf die Grenze der Lichtgeschwindigkeit abbremst.

Aber solche überlichtschnellen Geschwindigkeiten haben seltsame Zeiteigenschaften, Partikel reisen so schnell, dass sie ständig in der Zeit zurückspringen und immer aus der Zukunft kommen und in die Vergangenheit gehen. Von unserer Realität aus gesehen, könnte dies als Partikel angesehen werden, die ständig durch Wurmlöcher in die Vergangenheit reisen, und wir haben solche Partikel nie beobachtet, und es könnte nicht einmal möglich sein, solche Partikel zu beobachten.

Die Quantenverbindung wurde durch die Verwendung von Lasern und Quantenverschränkung als mindestens 10.000-mal schneller als Licht gemessen , sodass sie entweder unendlich oder nahezu unendlich schnell ist. Einstein mochte die Quantenverschränkung nicht, da sie seine Lichtgeschwindigkeitsbegrenzung zu überschreiten scheint. Wir wissen nicht, was die Quantenverbindung genau ist, einige schlagen Wurmlöcher vor, einige schlagen Superposition vor, einige sagen, es sei auf einer fundamentalen Ebene verbunden, teilchenfreie Wellen usw., aber wenn die Quantenverbindung reisende Teilchen beinhaltet, müssen diese Teilchen Tachyonen sein Teilchen mit unendlicher oder nahezu unendlicher Geschwindigkeit.

Und es ist seltsam, dass in der Quantenmechanik der Beobachter das Ergebnis eines Experiments beeinflussen kann. Das Ergebnis des Experiments liegt in der Zeit zurück, da es einige Nanosekunden dauert, bis das Licht uns erreicht, damit wir das Ergebnis des Experiments beobachten können. Da Beobachtung ein Ergebnis beeinflusst, ist Beobachtung so, als ob man vergangene Quantenpotentiale aus der Zukunft herauspickt? Und liegt das Experiment relativ gesehen wirklich in der Vergangenheit? Diese Dinge verwirren mich oft.

Richard Feynman und Wheeler hatten eine seltsame Hypothese, was wäre, wenn Licht von einem fortgeschrittenen Photon emittiert wird, das in der Zeit zurückreist und das Licht emittiert. Wir könnten argumentieren, dass dies die Kausalität nicht bricht, da für dieses Teilchen die lineare Zeit in die andere Richtung geht. Diese Antwort von Terry Bollinger erklärt diese Hypothese wunderbar: Kann ein Photon ohne Empfänger emittiert werden?

Inspiriert von Feynman frage ich mich, ob es hier eine Beziehung gibt, wenn ein Tachyon mit nahezu unendlicher Geschwindigkeit und nahezu null Energie von unserem Auge im selben Moment emittiert wird, in dem ein Licht empfangen wird, könnte das Tachyon in der Zeit zurückreisen zum Emitter und beeinflussen den Quantensprung, der das Licht aussendet? Die Beziehung ist dann: Stimmt die unendliche Geschwindigkeit mit der Lichtgeschwindigkeit in umgekehrter Richtung überein? Weiß jemand wie man das berechnet?

Wenn es eine solche Beziehung gibt, könnten wir uns fragen, ob die unendliche Geschwindigkeit eine Rolle bei der Festlegung der Lichtgeschwindigkeitsgrenze spielt.